Math Analysis Complete
Math Analysis Complete
2022
MATHEMATICAL ANALYSIS
Thanatyod Jampawai
Assistant Professor of Mathematics
Suan Sunandha Rajabhat University
2 Sequences in R 37
2.1 Limits of sequences . . . . . . . . . . . . . . . . . . . . . . . . . . . . . . . . . . . . 37
2.2 Limit theorems . . . . . . . . . . . . . . . . . . . . . . . . . . . . . . . . . . . . . . 47
2.3 Bolzano-Weierstrass Theorem . . . . . . . . . . . . . . . . . . . . . . . . . . . . . . 61
2.4 Cauchy sequences . . . . . . . . . . . . . . . . . . . . . . . . . . . . . . . . . . . . . 67
3 Topology on R 71
3.1 Open sets . . . . . . . . . . . . . . . . . . . . . . . . . . . . . . . . . . . . . . . . . 71
3.2 Closed sets . . . . . . . . . . . . . . . . . . . . . . . . . . . . . . . . . . . . . . . . . 76
3.3 Limit points . . . . . . . . . . . . . . . . . . . . . . . . . . . . . . . . . . . . . . . . 79
4 Limit of Functions 85
4.1 Limit of Functions . . . . . . . . . . . . . . . . . . . . . . . . . . . . . . . . . . . . 85
4.2 One-sided limit . . . . . . . . . . . . . . . . . . . . . . . . . . . . . . . . . . . . . . 94
4.3 Infinite limit . . . . . . . . . . . . . . . . . . . . . . . . . . . . . . . . . . . . . . . . 98
5 Continuity on R 103
5.1 Continuity . . . . . . . . . . . . . . . . . . . . . . . . . . . . . . . . . . . . . . . . . 103
5.2 Intermediate Value Theorem . . . . . . . . . . . . . . . . . . . . . . . . . . . . . . . 112
i
ii CONTENTS
6 Differentiability on R 121
6.1 The Derivative . . . . . . . . . . . . . . . . . . . . . . . . . . . . . . . . . . . . . . 121
6.2 Differentiability theorem . . . . . . . . . . . . . . . . . . . . . . . . . . . . . . . . . 127
6.3 Mean Value Theorem . . . . . . . . . . . . . . . . . . . . . . . . . . . . . . . . . . . 132
6.4 Monotone function . . . . . . . . . . . . . . . . . . . . . . . . . . . . . . . . . . . . 142
7 Integrability on R 149
7.1 Riemann integral . . . . . . . . . . . . . . . . . . . . . . . . . . . . . . . . . . . . . 149
7.2 Riemann sums . . . . . . . . . . . . . . . . . . . . . . . . . . . . . . . . . . . . . . . 165
7.3 Fundamental Theorem of Calculus . . . . . . . . . . . . . . . . . . . . . . . . . . . 174
FIELD AXIOMS.
There are functions + and ·, defined on R2 , that satisfy the following properties for every a, b, c ∈ R:
The real number system R contains certain special subsets: the set of natural numbers
N := {1, 2, 3, ...}
obtained by begining with 1 and successively adding 1’s to form 2 := 1 + 1, 3 := 2 + 1, etc,; the
set of integers
Z := {..., −2, −1, 0, 1, 2, ...}
(Zahlen is German for number); the set of rationals (or fractions or quoteints)
{ }
p
Q := : p, q ∈ Z and q ̸= 0
q
and the set of irrationals
Qc := R\Q.
Equality in Q is defined by
m p
= if and only if mq = np.
n q
Recall that each of the sets N, Z, Q and R is a proper subset of the next; i.e.,
N ⊂ Z ⊂ Q ⊂ R.
| · a {z
an = a · ... · a}
n− copies
1. (ab)n = an bn
( a )n an
2. = n where b ̸= 0
b b
3. an · am = am+n
an
4. = an−m where a ̸= 0
am
Proof. Excercise.
1.1. ORDERED FIELD AXIOMS 3
1. 0a = 0 3. −(−a) = a
0a = (0 + 0)a ( by F5 )
= 0a + 0a ( by F4 )
0 = 0a ( by 1. )
= (1 + (−1))a ( by F7 )
= 1a + (−1)a ( by F4 )
= a + (−1)a ( by F6 )
By F7, (−1)a is an additive inverse of a. Thus, (−1)a = −a. This result leds to
0 = a + (−a)
aa−1 = 1
0 = 0b ( by 1. in Theorem 1.1.4)
= (a + (−a))b ( by F7 )
= ab + (−a)b ( by F4 )
a=a+0 ( by F5 )
= a + (c + (−c)) ( by F7 )
= (a + c) + (−c) ( by F2 )
= (b + c) + (−c) ( by assumption )
= b + (c + (−c)) ( by F2 )
=b+0 ( by F7 )
=b ( by F5 )
a = a1 ( by F6 )
= a(cc−1 ) ( by F8 )
= (ac)c−1 ( by F2 )
= (bc)c−1 ( by assumption )
= b(cc−1 ) ( by F2 )
= b1 ( by F8 )
=b ( by F6 )
1.1. ORDERED FIELD AXIOMS 5
If ab = 0 , then a = 0 or b = 0.
Proof. Let a and b be real numbers. Suppose ab = 0 and a ̸= 0. By 1. in Theorem 1.1.4, we get
ab = 0 = a0
ORDER AXIOMS.
There is a relation < on R2 that has the following properties for every a, b, c ∈ R.
Example 1.1.8 Let x ∈ R. Show that if 0 < x < 1, then 0 < x2 < x
Proof. Let x be a real number such that 0 < x < 1. Then 0 < x and x < 1. By O4.1 and the fact
that x > 0, we obtain
Example 1.1.9 Let x, y ∈ R. Show that if 0 < x < y, then 0 < x2 < y 2
Proof. Let x and y be real numbers such that 0 < x < y. Then x > 0 and y > 0. By O4.1, we
obtain
0·x < x·x < y·x
0 < x2 < xy
and
0·y < x·y < y·y
0 < xy < y2.
Then 0 < x2 < xy and xy < y 2 . By Transitive Property, 0 < x2 < y 2 .
Proof. Let a, b, c and d be real numbers. Assume that a < b and c < d. By O3, we get
Proof. Let a, b, c and d be real numbers. Assume that 0 < a < b and 0 < c < d.
Then b > 0 and c > 0. By O4.1, we get
a ̸= 0 implies a2 > 0.
Proof. Let a be a real number. Assume that a ̸= 0. By Trichotomy Property (O1), a > 0 or a < 0.
Case a > 0. By O4.1, a · a > 0 · a. So, a2 > 0.
Case a < 0. By O4.2, a · a > 0 · a. So, a2 > 0.
Moreover, we see that 1 ̸= 0. So, 1 = 12 > 0. By cancellation for addition,
Proof. Let x ∈ R such that x > 0. Then x−1 ̸= 0. By Theorem 1.1.12, (x−1 )2 > 0. Thus,
Proof. Let x ∈ R such that x < 0. Then x−1 ̸= 0. By Theorem 1.1.12, (x−1 )2 > 0. Thus,
Theorem 1.1.15 Let a and b be real numbers such that 0 < a < b. Then
1 1
< .
b a
1
Proof. Let a and b be real numbers such that 0 < a < b. Then ab > 0. So, > 0.
ab
By O4.1, we obtain
1 1 1
0· < a· < b·
ab ab ab
1 1
0 < < .
b a
Example 1.1.16 Let x and y be two distinct real numbers. Prove that
x+y
lies between x and y.
2
Proof. Let x and y be two distinct real numbers.
By Trinochomy rule, x ̸= y. WLOG x < y. Then x + x < x + y and x + y < y + y.
By transitive rule,
2x < x + y < 2y
x+y
x< < y.
2
1.1. ORDERED FIELD AXIOMS 9
ABSOLUTE VALUE.
Definition 1.1.17 (Absolute Value) The absolute value of a number a ∈ R is a the number
a if a > 0
|a| = 0 if a = 0
−a if a < 0
Hence, |a| ≥ 0.
|ab| = |a||b|.
Case a > 0 and b > 0. Then ab > 0, |a| = a and |b| = b. So, |ab| = ab = |a||b|.
Case a > 0 and b < 0. Then ab < 0, |a| = a and |b| = −b. So, |ab| = −ab = a(−b) = |a||b|.
Case a < 0 and b > 0. Then ab < 0, |a| = −a and |b| = b. So, |ab| = −ab = (−a)b = |a||b|.
Case a < 0 and b < 0. Then ab > 0, |a| = −a and |b| = −b. So, |ab| = ab = (−a)(−b) = |a||b|.
|a − b| = |b − a|.
= | − 1|| − a + b| = 1 · | − a + b| = | − a + b| = |b − a|.
a≤M and −a ≤ M .
We obtain a ≥ −M . Thus, −M ≤ a ≤ M .
Conversely, assume that −M ≤ a ≤ M . Then
−M ≤ a and a ≤ M.
INTERVAL.
Let a and b real numbers. A closed interval is a set of the form
[a, b] := {x ∈ R : a ≤ x ≤ b} (−∞, b] := {x ∈ R : x ≤ b}
[a, ∞) := {x ∈ R : a ≤ x} (−∞, ∞) := R,
Notice, then, that when a < b, then intervals [a, b], [a, b), (a, b] and (a, b) correspond to line
segments on the real line, but when b < a, these interval are all the empty set.
12 CHAPTER 1. THE REAL NUMBER SYSTEM
0 < x < 2.
Solution. Let |x| < 1. Then −1 < x < 1. So, 0 < x + 1 < 2. We obtain
Therefore,
1
Example 1.1.27 Show that if |x − 1| < 2, then > 1.
|x|
Solution. Let |x − 2| < 1. Then −1 < x − 2 < 1. So, 1 < x < 3. We obtain
|x| > 1.
1
Therefore, > 1.
|x|
|a + b| ≤ |a| + |b|.
−|a| ≤ a ≤ |a|
−|b| ≤ b ≤ |b|
Proof. Let a, b ∈ R.
1. By Triangle Inequality,
2. By Triangle Inequality,
|a| = |(a − b) + b| ≤ |a − b| + |b|.
3. By 2,
|a| − |b| = |a| − | − b| ≤ |a − (−b)| = |a + b|.
4. By 2, |a| ≤ |a − b| + |b|. By 3,
Then,
|b| − |a − b| ≤ |a| ≤ |a − b| + |b|
−|a − b| ≤ |a| − |b| ≤ |a − b|
Thus, ||a| − |b|| ≤ |a − b|.
Proof. Let x, y ∈ R.
1. Assume that x < y + ε for all ε > 0 and x > y. Then x − y > 0. By assumption, we get
x < y + (x − y) = x.
It is imposible. So,x < y + ε for all ε > 0 if and only if x ≤ y Conversely, suppose that
there is an ε > 0 such that x ≥ y + ε. So,
x≥y+ε>y+0=y
2. Excercise.
Exercises 1.1
2.1 If a + b = a, then x = 0.
√
5. Prove that if 0 < a < 1 and b = 1 − 1 − a, then 0 < b < a.
√
6. Prove that if a > 2 and b = 1 − 1 − a, then 2 < b < a.
10. Prove that 0 < |x − 1| ≤ 1 implies |x3 + x − 2| < 8|x − 1|. Is this true if 0 ≤ |x − 1| < 1 ?
16 CHAPTER 1. THE REAL NUMBER SYSTEM
for all a1 , a2 , b1 , b2 ∈ R
15. Let x, y ∈ R. Prove that x > y − ε for all ε > 0 if and only if x ≥ y.
16. Suppose that x, a, y, b ∈ R, |x − a| < ε and |y − b| < ε for some ε > 0. Prove that
|a| + a
a+ :=
2
a+b
18. Let a, b ∈ R. The arithmetic mean of a, b is A(a, b) := ,
√ 2
the geometric mean of a, b ∈ (0, ∞) is G(a, b) := ab,
2
and harmonic mean of a, b ∈ (0, ∞) is H(a, b) := −1 .
a + b−1
Show that
S ⊆ N ∧ S ̸= ∅ → ∃m ∈ S ∀s ∈ S, m ≤ s.
Theorem 1.2.2 (Mathematical Induction) Suppose for each n ∈ N that P (n) is a statement
that satisfies the following two properties:
(2) Inductive step : For every k ∈ N for which P (k) is true, P (k + 1) is also true.
Proof. We will prove by contradiction. Assume that (1) and (2) are ture and there is an n0 ∈ N
such that P (n0 ) is false. Define
S = {n ∈ N : P (n) is false }.
∑
n
n(n + 1)
k=
k=1
2
for all n ∈ N.
∑
1
2 1(1 + 1)
Proof. For n = 1, we get k=1= = . So, (1) is true.
k=1
2 2
∑
n
n(n + 1)
Assume that k= . Then,
k=1
2
∑
n+1 ∑
n
n(n + 1) [n ] (n + 1)(n + 2)
k= k + (n + 1) = + (n + 1) = (n + 1) +1 = .
k=1 k=1
2 2 2
Proof. We will prove by induction on n. For n = 1, it is clear 21 > 1. Assume that 2n > n for
some n ∈ N. By inductive hypothesis and the fact that n ≥ 1,
2n+1 = 2n · 2 > 2n = n + n ≥ n + 1.
So, 2n > n is true for n + 1. We conclude by induction that 2n > n holds for n ∈ N.
BINOMIAL FORMULA.
n+1 n n
= +
k k−1 k
1.2. WELL-ORDERING PRINCIPLE 19
Proof. We will prove by induction on n. The formula is obvious for n = 1. Assume that the
formula is true for some n ∈ N. By inductive hypothesis,
∑ n
n
(a + b)n+1 = (a + b)(a + b)n = (a + b) an−k bk
k=0 k
∑n
n ∑n
n
= an−k+1 bk + an−k bk+1
k=0 k k=0 k
∑n
n ∑
n−1
n
= an+1 + an−k+1 bk + an−k bk+1 + bn+1
k=1 k k=0 k
∑n
n ∑n
n
= an+1 + an−k+1 bk + an−k+1 bk + bn+1
k=1 k k=1 k−1
∑n
n n
=a n+1
+ + an−k+1 bk + bn+1
k=1 k k−1
Thus, it follows from Theorem 1.2.6 that
∑n
n+1 ∑
n+1
n+1
(a + b)n+1 = an+1 + an−k−1 bk + bn+1 = an+1−k bk
k=1 k k=0 k
i.e., the formula is true for n + 1. We conclude by induction that the formula holds for n ∈ N.
20 CHAPTER 1. THE REAL NUMBER SYSTEM
Exercises 1.2
∑
n ∑
n
n(4n2 − 1)
1.1 (3k − 1)(3k + 2) = 3n + 6n + n
3 2
1.3 (2k − 1)2 =
k=1 k=1
3
∑n [ ]2 ∑n
n(n + 1) a−1 1
1.2 k3 = 1.4 = 1 − , a ̸= 0
k=1
2 k=1
ak an
8. Let 0 < |a| < 1. Prove that |a|n+1 < |a|n for all n ∈ N.
SUPREMUM.
a ≤ M for all a ∈ A
In this case we shall say that A has a supremum s and shall write s = sup A
A = (0, ∞) No ∅ None
A=N No ∅ None
A=Z No ∅ None
22 CHAPTER 1. THE REAL NUMBER SYSTEM
1. A = [0, 1] 2. A = (0, 1)
Solution.
M0 + 1 M0 + 1
But 0 < a < M0 < < 1, so belongs to A. It is imposible because M0 is
2 2
an upper bound of A. Hence, there is no upper bound of A such that it is less that 1. We
conclude that sup A = 1.
Theorem 1.3.4 If a set has one upper bound, then it has infinitely many upper bounds.
M := M0 + k for all k ∈ N.
Theorem 1.3.5 If a set has a supremum, then it has only one supremum.
Proof. Let s1 and s2 be suprema of the same of a set A. Then, s1 and s2 are upper bounds of A.
By definition of supremum, we obtain
s1 ≤ s2 and s2 ≤ s1 .
Therefore, s1 = s2 .
Proof. We will prove by contradiction. Assume that A has an infimum, say s. Suppose that there
a positve ε0 > 0 such that
s − ε0 < s
Proof. Assume that A ⊂ N has a supremum, say s. Apply APS to choose an x0 ∈ A such that
s − 1 < x0 ≤ s.
If x0 = s, then s ∈ A. In this case s − 1 < x0 < s. Apply again APS to choose x1 ∈ A such that
x0 < x1 < s
0 < x1 − x0 < s − x0 .
24 CHAPTER 1. THE REAL NUMBER SYSTEM
(s − 1) + x1 < x0 + s.
COMPLETENESS AXIOM.
If A is a nonempty subset of R that is bounded above, then A has a supremum.
Proof. Suppose that N is bounded above. Since N is not a nonempty set by Completeness Axiom,
N has a supremum, say s. Then
Theorem 1.3.9 (Archimedean Properties (AP)) For each x ∈ R, the following statements
are true.
Proof. Suppose that there is an x ∈ R such that x ≥ n for all n ∈ N. It means that x is an upper
bound of N. This is contradiction Theorem 1.3.8. Thus, part 1 is proved.
1 1
Next, we assume that x > 0. Then ∈ R. By 1, there is an n ∈ N such that < n. Thus,
x x
1
< x.
n
1
Proof. Let x ∈ R. Assume that |x| < for all n ∈ N. Let ε > 0. By AP, there an N ∈ N such
n
1
that < ε. By assumption, we obtain
N
1
|x| < < ε.
N
1
< 1 − u0 .
n0
1 1
Since n0 + 1 > n0 > 0, < . We obtain
n0 + 1 n0
1
< 1 − u0
n0 + 1
1 n0
u0 < 1 − =
n0 + 1 n0 + 1
n − 1 ≤ x < n.
n0 − 1 ≤ x < n 0 .
−m < x ≤ −m + 1.
If x = −m + 1, we choose n = −m + 2. So,
n − 1 = −m + 1 = x < n or n − 1 ≤ x < n.
If −m < x < −m+1, we choose n = −m+1. So, n−1 < x < n. It implies that n−1 ≤ x < n.
na + 1 < nb.
m
Set r := . We obtain a < r < b.
n
1.3. COMPLETENESS AXIOM 27
√
Theorem 1.3.15 2 is irrational.
√
Proof. Assume that 2 is a rational number. Then there are two integers p and q such that
√ p
2 = when q ̸= 0 and gcd(p, q) = 1.
q
We have 2q 2 = p2 . It implies that p is an even number. Then there is an k ∈ Z such that p = 2k.
So,
2q 2 = (2k)2 = 4k 2
q 2 = 2k 2
a b
Proof. Let a, b ∈ R such that a < b. Then √ < √ . By the Density of Rational, there is an
2 2
a b
r ∈ Q such that √ < r < √ . It follows that
2 2
√
a < r 2 < b.
√
If r ̸= 0, then t := r 2 is irrational (see Exercise). It is done.
a b
Case r = 0. By the Density of Rational, there is an s ∈ Q such that √ < 0 < s < √ . It follows
2 2
that
√
a < s 2 < b.
√
Set t = s 2, irrational. Thus, the proof is complete.
28 CHAPTER 1. THE REAL NUMBER SYSTEM
INFIMUM.
In this case we shall say that A has an infimum s and shall write ℓ = inf A
A = (−∞, 0) No ∅ None No
A=Z No ∅ None No
1.3. COMPLETENESS AXIOM 29
1. A = [0, 1] 2. A = (0, 1)
Solution.
m0 ≤ a for all a ∈ A
m0 m0
But 0 < < m0 ≤ a, so belongs to A. It is imposible because m0 is a lower bound
2 2
of A. Hence, there is no lower bound of A such that it is greater that 0. We conclude that
inf A = 0.
{ }
1
Example 1.3.20 Let A = : n ∈ N . Prove that inf A = 0.
n
1
Proof. For each n ∈ N, we get n > 0. So, > 0. Thus, 0 is a lower bound of A.
n
Suppose that that there is a lower bound m0 of A such that m0 > 0.
By AP, there is n0 ∈ N such that
1
< m0 .
n0
m0 ≤ a for all a ∈ A.
1 1
For n = 1, we have that = belongs to A.
2 1+1
1
m0 ≤ .
2
1
Therefore, inf A = .
2
Theorem 1.3.22 (Approximation Property for Infimum (API)) If A has an infimum and
ε > 0 is any positive number, then there is a point a ∈ A such that
Proof. Assume that A has an infimum, say ℓ0 . Suppose that there a positve ε0 > 0 such that
ℓ0 + ε0 > ℓ0
Exercises 1.3
5. If a set has one lower bound, then it has infinitely many lower bounds.
6. Prove that if A is a nonempty bounded subset of Z, then both sup A and inf A exist and
belong to A.
7. Prove that for each a ∈ R and each n ∈ N there exists a rational rn such that
1
|a − rn | < .
n
32 CHAPTER 1. THE REAL NUMBER SYSTEM
√ √
if a + b K = x + y K, then a = x and b = y.
10. Show that a lower bound of a set need not be unique but the infimum of a given set A is
unique.
11. Show that if A is a nonoempty subset of R that is bounded below, then A has a finite
infimum.
12. Prove that if x is an upper bound of a set A ⊆ R and x ∈ A, then x is the supremum of A.
13. Suppose E, A, B ⊂ R and E = A ∪ B. Prove that if E has a supremum and both A and B
are nonempty, then SupA and sup B both exist, and sup E is one of the numbers SupA or
sup B.
14. (Monotone Property) Suppose that A ⊆ B are nonempty subsets of R. Prove that
−A := {−x : x ∈ A}
15.1 A has a supremum if and only if −A has and infimum, in which case
inf(−A) = − sup A.
15.2 A has an infimum if and only if −A has and supremum, in which case
sup(−A) = − inf A.
1.4. FUNCTIONS AND INVERSE FUNCTIONS 33
if x1 = x2 , then y2 = y2 .
2x1 + 1 = 2x2 + 1
2x1 = 2x2
x1 = x2
y−1
So, f is 1-1. Let y ∈ R. Choose x = ∈ R. Then,
2
( )
y−1
f (x) = 2x + 1 = 2 +1=y
2
Theorem 1.4.3 Let X and Y be sets and f : X → Y . Then f is 1-1 from X onto Y if and only
if there is a unique function g from Y onto X that satisfies
1. f (g(y)) = y, y∈Y
and
Proof. Suppose that f is 1-1 and onto. For each y ∈ Y choose the unique x ∈ X such that
f (x) = y, and define
g(y) := x.
f (x) = f (g(y)) = y.
If f is 1-1 from a set X onto a set Y , we shall say that f has an inverse function. We shall
call the function g given in Theorem 1.4.3 the inverse of f , and denote it by f −1 . Then
f −1 (2x + 1) = f −1 (f (x)) = x
x−1
Substitue x := . We obtain
2
( )
−1 −1 x−1 x−1
f (x) = f 2· +1 = .
2 2
Example 1.4.5 Let f (x) = ex − e−x .
Solution. Let x1 , x2 ∈ R such that x1 ̸= x2 . WLOG x1 > x2 . Then ex1 > ex2 .
Since −x1 < −x2 , e−x1 < e−x2 . We obtain
Exercises 1.4
1. For each of the following, prove f is 1-1 from A onto A. Find a formula for f −1 .
3. Suppose that A is finite and f is 1-1 from A onto B. Prove that B is finite.
4. Prove that there a fuction f that is 1-1 from {2, 4, 6, ...} onto N.
5. Prove that there a fuction f that is 1-1 from {1, 3, 5, ...} onto N.
Sequences in R
1
0.9
0.8
0.7
0.6
0.5
0.4
0.3
0.2
0.1
X
1 2 3 4 5 6 7 8 9 10
Definition 2.1.3 A sequence of real numbers {xn } is said to converge to a real number a ∈ R
if and only if for every ε > 0 there is an N ∈ N such that
(c) lim xn = a;
n→∞
Y
a+ε
a
a−ε
X
1 2 3 ··· N N + 1 ···
Proof. Let k be a constant and ε > 0. We can choose whatever N ∈ N such that for each n ≥ N ,
we always obtain
|k − k| = 0 < ε.
So, lim k = k.
n→∞
1
Example 2.1.5 Prove that → 0 as n → ∞.
n
1
Proof. Let ε > 0. By AP, there is an N ∈ N such that < ε.
N
1 1
Let n ∈ N such that n ≥ N . Then ≤ . We obtain
n N
1 1 1
−0 = ≤ < ε.
n n N
1
Thus, → 0 as n → ∞.
n
2.1. LIMITS OF SEQUENCES 39
n
Example 2.1.6 Prove that lim =1
n→∞ n+1
1
Proof. Let ε > 0. By AP, there is an N ∈ N such that < ε.
N
1 1 1
Let n ∈ N such that n ≥ N . Then n + 1 > n ≥ N . So, < ≤ . We obtain
n+1 n N
n n − (n + 1) −1 1 1 1
−1 = = = < ≤ < ε.
n+1 n+1 n+1 n+1 n N
n
Thus, → 1 as n → ∞.
n+1
1
Example 2.1.7 Prove that → 0 as n → ∞
2n
1
Proof. Let ε > 0. By AP, there is an N ∈ N such that < ε.
N
1 1 1
Let n ∈ N such that n ≥ N . By Example 1.2.4, 2n > n. So, n < ≤ . We obtain
2 n N
1 1 1 1
− 0 = < ≤ < ε.
2n 2n n N
1
Thus, → 0 as n → ∞.
2n
1
Example 2.1.8 Prove that lim =0
n→∞ n2
√ 1 √
Proof. Let ε > 0. Then ε > 0. By AP, there is an N ∈ N such that < ε.
N
1 1
Let n ∈ N such that n ≥ N . Since n ≥ N > 0, n2 ≥ N 2 . Then 2 ≤ 2 . We obtain
n N
1 1 1
2
− 0 = 2 ≤ 2 < ε.
n n N
1
Thus, → 0 as n → ∞.
n2
(√ √ )
Example 2.1.9 Prove that lim n+1− n =0
n→∞
1
Proof. Let ε > 0. Then ε2 > 0. By AP, there is an N ∈ N such that < ε2 .
N
√ √ 1 1
Let n ∈ N such that n ≥ N . Since n ≥ N > 0, n ≥ N . Then √ ≤ √ .
n N
√ √ √ √ 1 1
Since n + 1 > 0, n + 1 + n > n. Then √ √ < √ . We obtain
n+1+ n n
√ √
√ √ √ √ n+1− n
n + 1 − n − 0 = ( n + 1 − n) · √ √
n+1− n
1 1 1
=√ √ < √ ≤ √ < ε.
n+1+ n n N
√ √
Thus, n + 1 − n → 0 as n → ∞.
40 CHAPTER 2. SEQUENCES IN R
2xn + 1 → 3 as n → ∞.
ε
|(2xn + 1) − 3| = |2(xn − 1)| = 2|xn − 1| < 2 · = ε.
2
Thus, 2xn + 1 → 3 as n → ∞.
(xn )2 → 1 as n → ∞.
Then, |xn | − |1| = |xn | − | − 1| ≤ |xn − (−1)| = |xn + 1| ≤ 1. So, |xn | < 2.
Let ε > 0. By assumption, there is an N2 ∈ N such that
ε
n ≥ N2 implies |xn + 1| < .
3
Let n ∈ N. Choose N = max{N1 , N2 }. For each n ≥ N , we obtain
Thus, (xn )2 → 1 as n → ∞.
2.1. LIMITS OF SEQUENCES 41
1
→ 1 as n → ∞.
xn
1 1 − xn 1 ε
−1 = ≤ · |xn − 1| < 2 · = ε.
xn xn |xn | 2
1
Thus, → 1 as n → ∞.
xn
1 + (xn )2
→ 1 as n → ∞
xn + 1
2 = |2 − xn + xn | = |1 − xn + 1 + xn | ≤ |1 − xn | + |1 + xn | ≤ 1 + |1 + xn |
1 ≤ |1 + xn |
1
≤ 1.
|1 + xn |
Proof. Assume that a sequence {xn } converges to both a and b. We will show that a = b by
Corollary 1.1.32. Let ε > 0. By assumption, there are N1 , N2 ∈ N such that
ε
n ≥ N1 implies |xn − a| <
2
and
ε
n ≥ N2 implies |xn − b| < .
2
Choose N = max{N1 , N2 }. For each n ≥ N , we obtain
ε ε
|a − b| = |(a − xn ) + (xn − b)| ≤ |xn − a| + |xn − b| < + = ε.
2 2
Hence, a − b = 0 or a = b. We conclude that the sequence {xn } can have at most one limit.
Example 2.1.15 Show that the limit {(−1)n }n∈N has no limit or does not exist (DNE).
SUBSEQUENCES.
Definition 2.1.16 By a subsequence of a sequence {xn }n∈N , we shall mean a sequence of the
form
Example 2.1.17 Give examples for two subsequences of the following sequences.
Sequences Subsequences
{n}n∈N 1, 3, 5, ...
2, 4, 6, ...
Theorem 2.1.18 If {xn }n∈N converges to a and {xnk }k∈N is any subsequence of {xn }n∈N , then
xnk converges to a as k → ∞.
nk ≥ k for all k ∈ N.
|xnk − a| < ε.
nk = 2k and nk = 2k − 1.
BOUNDED SEQUENCES.
Example 2.1.21 Show that the following sequence is bounded above or bounded below or bounded.
{n}n∈N Yes No No
1 ≤ n for all n ∈ N
{−n}n∈N No Yes No
−n ≤ 1 for all n ∈ N
Example 2.1.23 Show that the limit {n}n∈N does not exist.
Solution. Suppose that {n}n∈N converges. By BCT, there is a K > 0 such that
(xn )2 → 1 as n → ∞.
Exercises 2.1
1 5+n
1.1 3 + as n → ∞ 1.5 as n → ∞
n n2
( )
1 3
1.2 2 1 − as n → ∞ 1.6 π − √ as n → ∞
n n
2n + 1 n(n + 2)
1.3 as n → ∞ 1.7 as n → ∞
1−n n2 + 1
n2 − 1 n
1.4 as n → ∞ 1.8 3 as n → ∞
n2 n +1
2.1 2 − xn → 0 as n → ∞ 1
2.4 →1 as n → ∞
xn − 1
2.2 3xn + 1 → 7 as n → ∞
2 + x2n
2.3 (xn )2 + 1 → 5 as n → ∞ 2.5 →3 as n → ∞
xn
3. Assume that {xn } is a convergent sequence in R. Prove that lim (xn − xn+1 ) = 0.
n→∞
6. Prove that {(−1)n } has some subsequences that converge and others that do not converge.
8. Suppose that {bn } is a sequence of nonnegative numbers that converges to 0, and {xn } is a
real sequence that satisfies |xn − a| ≤ bn for large n. Prove that xn converges to a.
xn
9. Suppose that {xn } is bounded. Prove that → 0 as n → ∞ for all k ∈ N.
nk
10. Suppose that {xn } and {yn } converge to same point. Prove that xn − yn → 0 as n → ∞
Theorem 2.2.1 (Squeeze Theorem) Suppose that {xn }, {yn }, and {wn } are real sequences.
If xn → a and yn → a as n → ∞, and there is an N0 ∈ N such that
xn ≤ wn ≤ yn for all n ≥ N0 ,
then wn → a as n → ∞.
Proof. Let {xn }, {yn }, and {wn } be real sequences. Assume that xn → a and yn → a as n → ∞
and there is an N0 ∈ N such that
xn ≤ wn ≤ yn for all n ≥ N0 .
a − ε < xn ≤ wn ≤ yn < a + ε.
sin(n2 )
lim = 0.
n→∞ 2n
Theorem 2.2.3 Let {xn }, and {yn } be real sequences. If xn → 0 and {yn } is bounded, then
xn yn → 0 as n → ∞.
Proof. Let {xn }, and {yn } be real sequences. Assume that xn → 0 as n → ∞ and {yn } is bounded.
Then there is a K > 0 such that
ε
|xn yn − 0| = |xn ||yn | < · K = ε.
K
Hence, xn yn → 0 as n → ∞.
cos(1 + n)
Example 2.2.4 Show that lim = 0.
n→∞ n2
Solution. By the cosine fucntion property,
cos(1 + n)
By Theorem 2.2.3, we conclude that lim = 0.
n→∞ n2
2.2. LIMIT THEOREMS 49
xn → sup A as n → ∞.
xn → inf A as n → ∞.
Proof. Exercise for 1. We will prove 2. Suppose A has a finite infimum. By API, there is x ∈ A
such that
1
inf A ≤ xn < inf A +
n
Theorem 2.2.6 (Additive Property) Suppose that {xn } and {yn } are real sequences.
If {xn } and {yn } are convergent, then
ε ε
|(xn + yn ) − (a + b)| = |(xn − a) + (yn − b)| ≤ |xn − a| + |yn − b| < + = ε.
2 2
ε |α|
|αxn − αx| = |α||xn − x| < |α| · = ε < 1 · ε = ε.
|α| + 1 |α| + 1
Theorem 2.2.8 (Multiplicative Property) Suppose that {xn } and {yn } are convergent se-
quences. Then
( )( )
lim (xn yn ) = lim xn lim yn .
n→∞ n→∞ n→∞
|xn yn − ab| = |xn (yn − b) + (xn − a)b| ≤ |xn ||yn − b| + |xn − a||b|
ε ε ε ε |b| ε ε
<K· + |b| = + · < + · 1 = ε.
2K 2(|b| + 1) 2 2 (|b| + 1) 2 2
1 1
lim =
n→∞ xn lim xn
n→∞
|a| |a|
Then |a| = |a − xn + xn | ≤ |xn − a| + |xn | ≤ + |xn |. So, ≤ |xn |, i.e.,
2 2
1 2
≤ .
|xn | |a|
1 1 a − xn 1 |xn − a| 2 |a|2
− = ≤ · < · ε = ε.
xn a axn |xn | |a| |a| 2|a|
1 1
Therefore, lim = .
n→∞ xn lim xn
n→∞
Theorem 2.2.10 (Quotient Property) Suppose that {xn } and {yn } are convergent sequences.
Then
xn lim xn
lim = n→∞
n→∞ yn lim yn
n→∞
Proof. The proof of Theorem is result from Multiplicative Property and Reciprocal Property.
n2 + n − 3
Example 2.2.11 Find the limit lim .
n→∞ 1 + 3n2
Solution.
n2 + n − 3 n2 (1 + n1 − n32 )
lim = lim
n→∞ 1 + 3n2 n→∞ n2 ( n12 + 3)
1+ − n32
1
n
= lim 1
n→∞
n2
+3
1 1
lim 1 + lim − 3 lim 2
n→∞ n→∞ n n→∞ n
=
1
lim 2 + lim 3
n→∞ n n→∞
1 + 0 − 3(0)
=
0+3
1
= .
3
2.2. LIMIT THEOREMS 53
Theorem 2.2.12 (Comparison Theorem) Suppose that {xn } and {yn } are convergent se-
quences. If there is an N0 ∈ N such that
xn ≤ yn for all n ≥ N0 ,
then
lim xn ≤ lim yn .
n→∞ n→∞
In particular, if xn ∈ [a, b] converges to some point c, then c must belong to [a, b].
xn ≤ yn for all n ≥ N0 .
Suppose that lim xn > lim yn , i.e., a > b. Then a − b > 0. By assumption, there is an N1 , N2 ∈ N
n→∞ n→∞
such that
a−b
n ≥ N1 implies |xn − a| <
2
and
a−b
n ≥ N2 implies |yn − b| < .
2
For each n ≥ max{N0 , N1 , N2 }, it follows that
a−b a−b
yn < b + =a− < xn
2 2
DIVERGENT.
n ≥ N > M.
n2
Example 2.2.15 Prove that lim = +∞.
n→∞ 1 + n
n2 n2 − 1 (n − 1)(n + 1)
> = = n − 1 > N − 1 > M.
1+n 1+n 1+n
n2
Hence, lim = +∞.
n→∞ 1 + n
2.2. LIMIT THEOREMS 55
4n2
Example 2.2.16 Prove that lim = −∞.
n→∞ 1 − 2n
−1 − 2N < M.
Let n ∈ N such that n ≥ N . It is clear that 2n − 1 > 0 and −2n < −2N . Since 0 < 1,
We obtain
4n2
Therefore, lim = −∞.
n→∞ 1 − 2n
Theorem 2.2.18 Let {xn } and {yn } be a real sequence and xn ̸= 0. If {yn } is bounded and
xn → +∞ or xn → −∞ as n → ∞, then
yn
lim = 0.
n→∞ xn
sin n
Example 2.2.19 Show that → 0 as n → ∞.
n
Solution. By property of sine, we have
M
αxn > α · = M.
α
Theorem 2.2.21 Let {xn } and {yn } be real sequences. Suppose that {yn } is bounded below and
xn → +∞ as n → ∞. Then
lim (xn + yn ) = +∞.
n→∞
m ≤ yn for all n ∈ N.
xn + yn > (M − m) + m = M.
Theorem 2.2.22 Let {xn } and {yn } be real sequences such that
It follows that
1. Exercise.
2. Assume that xn → −∞ as n → ∞. Let M ∈ R.
Case M = 0. There is an N ∈ N such that
xn · yn < 0 = M.
M M M
xn · yn < − · yn = · (−yn ) < · (−K) = −M < 0 < M.
K K K
Case M < 0. There is an N ∈ N such that
M
n≥N implies xn < < 0.
K
Let n ∈ N such that n ≥ N . Since yn > K > 0, −yn < −K < 0. We obtain
M −M −M
xn · yn < · yn = · (−yn ) < · (−K) = M.
K K K
Thus, lim xn yn = −∞.
n→∞
2.2. LIMIT THEOREMS 59
Exercises 2.2
sin(n4 + n + 1) n
1.1 xn = 1.4 xn =
n 2n
n (−1)n
1.2 xn = 2 1.5 xn =
n +1 n
√
n+1 1 + (−1)n
1.3 xn = 1.6 xn =
n+1 2n
√
2n(n + 1) 2n2 − 1
2.1 xn = 2.4 xn =
n2 + 1 n+1
1 + n − 3n2 √ √
2.2 xn =
3 − 2n + n2 2.5 xn = n + 2 − n
n3 + n + 5 √
2.3 xn = 3 2.6 xn = n2 + n − n
5n + n − 1
3.1 xn = n2 n2 + 1
3.4 xn =
n+1
3.2 xn = −n 1 − n2
3.5 xn =
n
n 2n
3.3 xn = √ 3.6 xn =
1+ n n
xn → sup A as n → ∞.
√
6.1 Suppose that 0 ≤ x1 ≤ 1 and xn+1 = 1 − 1 − xn for n ∈ N.
If xn → x as n → ∞, prove that x = 0 or 1.
60 CHAPTER 2. SEQUENCES IN R
√
6.2 Suppose that x1 > 0 and xn+1 = 2 + xn for n ∈ N.
If xn → x as n → ∞, prove that x = 2.
7. Let {xn } be a real sequence and α > 0. If xn → −∞ as n → ∞, then lim (αxn ) = −∞.
n→∞
8. Let {xn } and {yn } be real sequences such that yn > K for some K > 0 and all n ∈ N.
Prove that if xn → −∞ as n → ∞, then lim (xn yn ) = −∞.
n→∞
9. Let {xn } and {yn } are real sequences. Suuppose that {yn } is bounded above and xn → −∞
as n → ∞. Prove that
lim (xn + yn ) = −∞.
n→∞
Prove that
MONOTONE.
Example 2.3.2 Determine whether {xn }n∈N is increasing or decreasing or NOT both.
{n}n∈N Yes No No
1 ≤ 2 ≤ 3 ≤ ...
No Yes No
{ }
1
1≥ 1
2
≥ 1
3
≥ ...
n n∈N
1 ≤ 1 ≤ 1 ≤ ... 1 ≥ 1 ≥ 1 ≥ ...
{(−1)n }n∈N No No No
−1 ≤ 1 ≥ −1 ≤ 1 ≥ ... −1 ≤ 1 ≥ −1 ≤ 1 ≥ ...
62 CHAPTER 2. SEQUENCES IN R
Proof. Assume that {xn } is increasing and bounded above. By the Completeness Axiom, the
supremum
Definition 2.3.6 A sequence of sets {In }n∈N is said to be nested if and only if
1 1
1≤x≤ < .
n+1 n
Theorem 2.3.8 (Nested Interval Property) If {In }n∈N is a nested sequence of nonempty closed
bounded intervals, then
∩
E= In := {x : x ∈ In for all n ∈ N}
n∈N
contains at least one number. Moreover, if the lengths of these intervals satisfy |In | → 0 as n → ∞,
then E contains exactly one number.
We obtain a1 ≤ a2 ≤ a3 ≤ ... and b1 ≥ b2 ≥ b3 ≥ ... So, {an } is increasing and bounded above by
a1 and {bn } is decreasing bounded below by b1 . By MCT, there are a and b such that an → a and
bn → b as n → ∞.
a1 a2 a3 ... a b ... b3 b2 b1
Since an ≤ bn for all n ∈ N, it also follows from the Comparison Theorem that
an ≤ a ≤ b ≤ bn .
Hence, a number x belongs to In for all n ∈ N if and only if a ≤ x ≤ b. We obtain E = [a, b].
Suppose that |In | → 0 as n → ∞. Then bn − an → 0 as n → ∞, and we have by Addition
Property that a − b = 0. In particular, E = [a, a] = {a} contain exactly one number.
64 CHAPTER 2. SEQUENCES IN R
Theorem 2.3.9 (Bolzano-Weierstrass Theorem) Every bounded sequence of real numbers has
a convergence subsequence.
b−a
|Ik | = , xnk ∈ Ik , and xn ∈ Ik for infinitely many n. (2.2)
2k
Exercises 2.3
1. Prove that
(n2 + 22n + 65) sin(n3 )
xn =
n2 + n + 1
has a convergence sunsequence.
2. If {xn } is decreasing and bounded below, then {xn } has a finite limit.
4. Suppose that {xn } is a monotone increasing in R (not necessarily bounded above). Prove
that there is extended real number x such that xn → x as n → ∞.
√
5. Suppose that 0 < x1 < 1 and xn+1 = 1 − 1 − xn for n ∈ N. Prove that
xn+1 1
xn ↓ 0 as n → ∞ and → , as n → ∞
xn 2
1 n+1
6. If a > 0, prove that a n → 1 as n → ∞. Use the resulte to find the limit of {3 n }.
√
7. Let 0 ≤ x1 ≤ 3 and xn+1 = 2xn + 3 for n ∈ N. Prove that xn ↑ 3 as n → ∞.
√
8. Suppose that x1 ≥ 2 and xn+1 = 1 + xn − 1 for n ∈ N. Prove that xn ↓ 2 as n → ∞. What
happens when 1 ≤ x1 < 2 ?
9. Prove that
1 if x > 0
1
lim x 2n−1 = 0 if x = 0
n→∞
−1 if x < 0
1 + xn−1
10. Suppose that x0 ∈ R and xn = for n ∈ N. Prove that xn → 1 as n → ∞.
2
11. Let {xn } be a sequence in R. Prove that
xn + yn √
xn+1 = and yn+1 = xn yn , for n ∈ N
2
12.2 Prove that yn is increasing and bounded above, and xn is decreasing and bounded below.
x1 − y1
12.3 Prove that 0 < xn+1 − yn+1 < for n ∈ N
2n
12.4 Prove that lim xn = lim yn . (the common value is called the arithemetic-geometric
n→∞ n→∞
mean of x1 and y1 .)
and
yn = xn−1 + yn−1
xn √
→ 2 as n → ∞.
yn
√
14. (Archimedes) Suppose that x0 = 2 3, y0 = 3,
2xn−1 yn−1 √
xn = , and yn = xn yn−1 for n ∈ N.
xn−1 + yn−1
Definition 2.4.1 A sequence of points xn ∈ R is said to be Cauchy if and only if every ε > 0
there is an N ∈ N such that
1 1 1 1 1 1 2
− ≤ + ≤ + = < ε.
n m n m N N N
{ }
1
Thus, is Cauchy.
n
Proof. Let {xn } and {yn } be Cauchy. Let ε > 0. There are N1 , N2 ∈ N such that
ε
m, n ≥ N1 imply |xn − xm | <
2
and
ε
m, n ≥ N2 imply |yn − ym | < .
2
≤ |xn − xm | + |yn − ym |
ε ε
< + = ε.
2 2
Theorem 2.4.5 (Cauchy’s Theorem) Let {xn } be a sequence of real numbers. Then
Example 2.4.6 Prove that any real sequence {xn } that satisfies
1
|xn − xn+1 | ≤ , n ∈ N,
2n
is convergent.
1
Proof. Let ε > 0. By AP, there is an N ∈ N such that < ε.
N
1 1
Let n, m ∈ N such that n, m ≥ N . Then ≤ . By the fact that n < 2n for all n ∈ N,
n N
1 1 1
we get n < . Suppose that m > n. Then m − n > 0. So, 1 − m−n ≤ 1. We obtain
2 n 2
Exercises 2.4
1 n
1.1 xn = 1.2 xn =
n2 n+1
1+n
|xn | ≤
1 + n + 2n2
4. Suppose that xn ∈ N for n ∈ N. If {xn } is Cauchy prove that there are numbers a and N
such that xn = a for all n ≥ N .
1
if n, m ≥ N , then |xn − xm | < for all k ∈ N.
k
6. Let {xn } be Cauchy. Prove that {xn } converges if and only if at least one of its subsequence
converges.
∑
n
(−1)k
7. Prove that lim exists and is finite.
n→∞
k=1
k
|xk+1 − xk | ≤ a−k
Topology on R
Open sets are among the most important subsets of R. A collection of open sets is called a
topology, and any property (such as convergence, compactness, or continuity) that can be dened
entirely in terms of open sets is called a topological property.
Definition 3.1.1 A set E ⊆ R is open if for every x ∈ E there exists a δ > 0 such that
(x − δ, x + δ) ⊆ E.
In other word,
E is open ↔ ∀x ∈ E ∃δ > 0, (x − δ, x + δ) ⊆ E
and
Since the empty set has no element, by definition it imples that ∅ is open. For E = R, we
obatin
∀x ∈ R ∃δ > 0, (x − δ, x + δ) ⊆ R is true.
0 x 1
x
Theorem 3.1.3 Intervals (a, b), (a, ∞) and (−∞, b) are open.
{ }
x−a b−x
Proof. 1. Let x ∈ (a, b). Choose δ = min , . We obtain (x − δ, x + δ) ⊆ (a, b).
2 2
Hence, (a, b) is open.
x−a
2. Let x ∈ (a, ∞). Choose δ = . We obtain (x − δ, x + δ) ⊆ (a, ∞). Hence, (a, ∞) is open.
2
b−x
3. Let x ∈ (−∞, b). Choose δ = . We obtain (x − δ, x + δ) ⊆ (−∞, b). Hence, (−∞, b) is
2
open.
Proof. Suppose that [0, 1) is open. Given x = 0, there is a δ > 0 such that
Since −δ < − 2δ < 0, − 2δ ∈ (−δ, δ). It implies that − 2δ ∈ [0, 1) which is imposible.
3.1. OPEN SETS 73
Theorem 3.1.5 Let A and B be open. Prove that A ∪ B and A ∩ B are open.
(x − δ1 , x + δ1 ) ⊆ A and (x − δ2 , x + δ2 ) ⊆ B.
∩
n
2. Ak := A1 ∩ A2 ∩ ... ∩ An is open.
k=1
Proof. Excercise
74 CHAPTER 3. TOPOLOGY ON R
NEIGHBORHOOD.
Next, we introduce the notion of the neighborhood of a point, which often gives clearer, but
equivalent, descriptions of topological concepts than ones that use open intervals.
Proof. If every x ∈ E has a neighborhood U such that U ⊆ E, then there is a δ > 0 such that
(x − δ, x + δ) ⊆ U ⊆ E.
Hence, E ⊆ R is open .
Theorem 3.1.9 A sequence {xn } of real numbers converges to a limit x ∈ R if and only if for
every neighborhood U of x there exists N ∈ N such that xn ∈ U for all n > N .
(x − ε, x + ε) ⊆ U.
Therefore, xn → x as n → ∞.
3.1. OPEN SETS 75
Exercises 3.1
1. Show that interval [a, b], [a, b) and (a, b], are not open.
∩
∞
In+1 ⊂ In for each n ∈ N and In = ∅.
n=1
76 CHAPTER 3. TOPOLOGY ON R
F c = R\F = {x ∈ R : x ∈
/ F } is open.
Solution. Consider [0, 1]c = (−∞, 0) ∪ (1, ∞). By Theorem 3.1.3 and 3.1.5, we obtain
So, (−∞, 0) ∪ [1, ∞) is not open. We conclude that [0, 1) is neither open nor closed.
Theorem 3.2.4 Let A and B be closed. Prove that A ∪ B and A ∩ B are closed.
Proof. Let A and B be closed. Then Ac and B c are open. By Theorem 3.1.5, it implies that
∩
n
2. Ak := A1 ∩ A2 ∩ ... ∩ An is closed.
k=1
Proof. Let A1 , A2 , .., An be closed sets. Then Ac1 , Ac2 , .., Acn are open. We consider
( n )c
∪
Ak = (A1 ∪ A2 ∪ ... ∪ An )c = Ac1 ∩ Ac2 ∩ ... ∩ Acn
(k=1 )c
∩n
Ak = (A1 ∩ A2 ∩ ... ∩ An )c = Ac1 ∪ Ac2 ∪ ... ∪ Acn
k=1
Exercises 3.2
1. Show that interval [a, b], [a, ∞) and (−∞, b] are closed.
∪∞ [ ]
1 n−1
5. Is , open or closed ?
n=1
n n
∩
∞
show that In = [a, b].
n=1
∩
∞
7. Find a sequence In of closed interval that In+1 ⊂ In for each n ∈ N and In = ∅.
n=1
(
Jx = x − ε, x + δ),
where the union is taken over all ε > 0 and δ > 0 such that (x − ε, x + δ) ⊂ U .
Definition 3.3.1 A point x ∈ R is called a limit point of a set A ⊆ R if for every ε > 0 there
exists a ∈ A, a ̸= x, such that a ∈ (x − ε, x + ε) or
( )
x
( )
x
( )
x
A
80 CHAPTER 3. TOPOLOGY ON R
Set Set of limit points Set of interior points Set of isolated points
{1} ∅ ∅ {1}
N ∅ ∅ N
Q R R ∅
We obtain
[(−ε, 0) ∪ (0, ε)] ∩ (0, 1) ̸= ∅.
Proof. Let A and B be sets such that A ⊆ B. Let x ∈ A′ . Then, for all ε > 0, we obtain
Since A ⊆ B,
(x − ε, x + ε) ⊆ Ac .
CLOSURE.
{1} ∅ {1}
N ∅ N
Q R R
Proof. Let A and B be sets such that A ⊆ B. By Theorem 3.3.6, it implies that A′ ⊆ B ′ .
We conclude that Ā = A ∪ A′ ⊆ B ∪ B ′ = B̄.
Since x ∈
/ A, (x − ε, x + ε) ∩ A = [(x − ε, x) ∪ (x, x + ε)] ∩ A. Use the fact that A ⊆ Ā, we obtain
(x − ε, x + ε) ∩ Ā = ∅.
Ā = A ∪ A′ ⊆ A.
Proof. Let F be a closed set. Assume that {xn } is a sequence in F . We will prove by contradiction.
Assume that xn → a as n → ∞ and a ∈
/ F . Then a ∈ F c .
Since F c is open, there δ > 0 such that (a − δ, a + δ) ⊆ F c . So,
(a − δ, a + δ) ∩ F = ∅ (3.1)
Exercises 3.3
1. Identify the limit points, interior point and isolated points of the following sets:
4.1 (A ∩ B)′ = A′ ∩ B ′
4.2 A ∪ B = Ā ∪ B̄
4.3 A ∩ B = Ā ∩ B̄
4.4 (A ∪ B)◦ = A◦ ∪ B ◦
4.5 (A ∩ B)◦ = A◦ ∩ B ◦
7. Suppose x is a limit point of the set A. Show that for every ε > 0, the set
(x − ε, x + ε) ∩ A is infinite.
∪
∞ ∪
∞
8. Suppose that Ak ⊆ R for each k ∈ N, and let B = Ak . Show that B̄ = Āk .
k=1 k=1
Limit of Functions
Y
y = f (x)
L+ε
L
L−ε
X
a−δ a a+δ
86 CHAPTER 4. LIMIT OF FUNCTIONS
lim f (x) = 3.
x→1
ε
Proof. Let ε > 0. Choose δ = > 0. Let x ∈ R such that 0 < |x − 1| < δ. We obtain
2
Thus, f (x) → 3 as x → 1.
√
Example 4.1.3 Let f (x) = x2 where x ∈ R. Prove that f (x) → 0 as x → 0.
Proof. Let ε > 0. Choose δ = ε > 0. Let x ∈ R such that 0 < |x| < δ. We obtain
√
|f (x) − 0| = | x2 − 0| = |x| < ε.
√
Thus, x2 → 0 as x → 0.
ε
|x2 − 9| = |(x + 3)(x − 3)| = |x + 3||x − 3| ≤ (|x| + |3|)δ < (4 + 3) = ε.
7
√
Thus, x → 0 as x → 0.
1
Example 4.1.6 Prove that f (x) = → 1 as x → 1.
x
{ }
1 ε
Proof. Let ε > 0. Choose δ = min , . Let x ∈ R\{0} such that 0 < |x − 1| < δ.
2 2
1
Then 0 < |x − 1| < . By Triangle inequality,
2
1
1 = |1 − x + x| ≤ |1 − x| + |x| < + |x|.
2
1 1
So, |x| > . It follows that < 2. We obtain
2 |x|
1 1−x 1 ε
−1 = = · |x − 1| < 2δ < 2 · = ε.
x x |x| 2
1
Thus, f (x) → as x → 1.
x
Theorem 4.1.7 (Limit of Constant function) The limit of a constant function is equal to the
constant.
Theorem 4.1.8 (Limit of Linear function) Let m and c be constant such that f (x) = mx + c
for all x ∈ R. Then
lim (mx + c) = ma + c.
x→a
ε
Proof. Let ε > 0. Choose δ = > 0. Let x ∈ R such that 0 < |x − a| < δ.
|m| + 1
|m|
We obtain by < 1 that
|m| + 1
Proof. Assume that f (x) = g(x) for all x ∈ E\{a} and f (x) → L as x → a.
Let ε > 0. There is a δ > 0.
From 0 < |x − a| < δ, it implies that x ̸= a. So, f (x) = g(x) on the condition. We obtain
Thus, g(x) → L as x → a.
x2 − 1
Example 4.1.10 Prove that f (x) = has a limit as x → 1.
x−1
Solution. We see that g(x) = x + 1. We have
x2 − 1 (x − 1)(x + 1)
f (x) = = = x + 1 = g(x) for all x ̸= 1
x−1 x−1
By Theorem 4.1.9, it follows that
x2 − 1
lim = lim f (x) = lim g(x) = lim (x + 1) = 2.
x→1 x − 1 x→1 x→1 x→1
4.1. LIMIT OF FUNCTIONS 89
Proof. Assume that the limit of f (x) exists and equals to L and assume that a sequence xn ∈ E\{a}
that converges to a as n → ∞. Let ε > 0. There is a δ > 0 such that for all x ∈ E,
Coversely, assume that f (xn ) → L as n → ∞ for every sequence xn ∈ E\{a} that converges
to a as n → ∞. Suppose that f (x) does not converge to L as x approaches to a.
There is an ε0 > 0 such that
1 1
Choose δ = for all n ∈ N. Then 0 < |x − a| < . By Squeeze Theorem, xn → a as n → ∞.
n n
By assumption, f (xn ) → L as n → ∞, i.e., there N ∈ N
has no limit as x → 0.
Then f (xn ) and f (yn ) converge to distinct limits. By SCL, we conclude that f has no limit as
x → 0.
Next, we will use the SCL together Theorems of limit for addition, mutiplication, scalar mul-
tiplication and quotient in order to proof Theorem 4.1.13.
In fact,
( ) lim f (x)
f
4. lim (x) = x→a when the limit of g(x) is nonzero.
x→a g lim g(x)
x→a
Proof. Exercise.
and lim g(x) = lim h(x) = L, then the limit of f (x) exists, as x → a and
x→a x→a
lim f (x) = L.
x→a
Corollary 4.1.17 Suppose that E ⊆ R and f, g : E → R are functions. Let a ∈ R be a limit point
of E and M > 0. If
then
lim f (x)g(x) = 0.
x→a
Proof. Assume that |g(x)| ≤ M for all x ∈ E\{a} and lim f (x) = 0.
x→a
Case f (x) = 0. Then f (x)g(x) = 0. It follows that lim f (x)g(x) = 0.
x→a
Case f (x) ̸= 0. Then |f (x)| > 0. So, lim M |f (x)| = 0. We obtain
x→a
By the Squeeze Theorem for Functions, it imlies that lim |g(x)f (x)| = 0.
x→a
From Theorem 4.1.15, we conclude that lim f (x)g(x) = 0.
x→a
92 CHAPTER 4. LIMIT OF FUNCTIONS
( )
1
Example 4.1.18 Show that lim x cos =0
x→0 x
Solution. By property of sine,
( )
1
cos ≤ 1 for all x ̸= 0.
x
( )
1
We have lim x = 0. By Corollary 4.1.17, lim x cos = 0.
x→0 x→0 x
then
lim f (x) ≤ lim g(x).
x→a x→a
Proof. Use SCL together the Comparison Theorem (Theorem 2.2.12), we will this theorem.
4.1. LIMIT OF FUNCTIONS 93
Exercises 4.1
1. Use Definition 4.1.1, prove that each of the following limit exists.
x−1
1.2 lim x2 − x + 1 = 3 1.4 lim = −1
x→2 x→0 x + 1
9.2 Show that there is a function such that as x → a, |f (x)| → |L| but the limit of f (x)
does not exist.
94 CHAPTER 4. LIMIT OF FUNCTIONS
√
What is the limit of f (x) := x − 1 as x → 1.
4
√
3 y= x−1
2
1
X
0 1 2 3 4 5 6 7 8 9
A reasonable answer is that the limit is zero. This function, however, does not satisfy Definition
4.1.1 because it is not defined on an OPEN interval containg a = 1. Indeed, f is defined only for
x ≥ 1. To handle such situations, we introduce one-sided limits.
1. A real function f said to converge to L as x approaches a from the right if and only if
f defined on some interval I with left endpoint a and every ε > 0 there is a δ > 0 such that
a + δ ∈ I and for all x ∈ I,
Y
y = f (x)
L+ε
X
a a+δ
4.2. ONE-SIDED LIMIT 95
2. A real function f said to converge to L as x approaches a from the left if and only if f
defined on some interval I with right endpoint a and every ε > 0 there is a δ > 0 such that
a + δ ∈ I and for all x ∈ I,
Y
y = f (x)
L
L−ε
X
a−δ a
√
Example 4.2.2 Prove that lim+ x − 1 = 0.
x→1
Proof. Let ε > 0. Choose δ = ε2 > 0. Let x > 1 such that 0 < x − 1 < δ. We obtain
√ √ √
|f (x) − 0| = | x − 1 − 0| = x − 1 < δ = ε.
√
Thus, x − 1 → 0 as x → 1+ .
|x|
Example 4.2.3 If f (x) = , prove that f has one-sided limit at a = 0 but lim f (x) = 0 DNE.
x x→0
Solution. Let ε > 0. We can choose any δ > 0. Let x ∈ R\{0} such that −δ < x < 0.
Then |x| = −x. We obtain
|x| −x
|f (x) − 0| = − (−1) = − (−1) = | − 1 + 1| = 0 < ε.
x x
Thus, lim− f (x) = −1. Similarly, lim− f (x) exists and equals 1.
x→0 x→0
Choose two sequence as follow
1
xn = →0 and f (xn ) = 1 → 1,
n
1
yn = − → 0 and f (yn ) = −1 → −1.
n
Then f (xn ) and f (yn ) converge to distinct limits. By SCL, we conclude that f has no limit as
x → 0.
96 CHAPTER 4. LIMIT OF FUNCTIONS
lim f (x)
x→a
Proof. Assume that f (x) → L as x → a. Let ε > 0. There is a δ > 0 such that
If a < x < a + δ, it satisfies (4.3) which implies |f (x) − L| < ε. Thus, lim+ f (x) = L.
x→a
If a − δ < x < a, it satisfies (4.3) which implies |f (x) − L| < ε. Thus, lim− f (x) = L.
x→a
Conversely, assume that L = lim+ f (x) = lim− f (x). Let ε > 0. There are δ1 , δ2 > 0 such that
x→a x→a
and
|f (x) − L| < ε.
Exercises 4.2
1. Use definitons to prove that lim+ f (x) exists and equal to L in each of the following cases.
x→a
2. Use definitons to rove that lim− f (x) exists and equal to L in each of the following cases.
x→a
x3 − 3x + 2 cos x
3.2 lim− 3.4 lim
x→1 x3 − 1
π−
x→ 2 1 − sin x
√ √
1 − cos x 2
4. Prove that → as x → 0+ .
sin x 2
5. Determine whether the following functions are limit at a.
3x + 1 if x ≥ 1
5.1 f (x) = and a=1
x + 3 if x < 1
2 − 2x if x ≥ 0
5.2 f (x) = √ and a=0
1−x if x < 0
6. Suppose that f : [0, 1] → R and f (a) = lim f (x) for all x ∈ [0, 1]. Prove that
x→a
f (q) = 0 for all q ∈ Q ∩ [0, 1] if and only if f (x) = 0 for all x ∈ [0, 1].
98 CHAPTER 4. LIMIT OF FUNCTIONS
The definition of limit of real functions can be expanded to include extended real numbers.
1. We say that f (x) → L as x → ∞ if and only if there exists a c > 0 such that (c, ∞) ⊆ E
and for every ε > 0, there is an M ∈ R such that
2. We say that f (x) → L as x → −∞ if and only if there exists a c > 0 such that (−∞, −c) ⊆ E
and for every ε > 0, there is an M ∈ R such that
1
Example 4.3.2 Prove that lim = 0.
x→∞ x
1
Proof. Let ε > 0. Choose M = > 0. If x > M > 0, it implies
ε
1 1 1
−0 = < = ε.
x x M
1
We conclude that lim = 0.
x→∞ x
x−1
Example 4.3.3 Prove that lim exists and equals to 1.
x→∞ x + 1
2
Proof. Let ε > 0. Choose M = > 0. If x > M > 0, it follows that x + 1 > x > M . So,
ε
1 1
< . We obtain
x+1 M
x−1 −2 1 2
−1 = =2· < = ε.
x+1 x+1 x+1 M
x−1
We conclude that lim = 1.
x→∞ x + 1
4.3. INFINITE LIMIT 99
1
Example 4.3.4 Prove that lim = 0.
x→∞ x2 +1
1
Proof. Let ε > 0. Choose M = √ > 0. If x > M > 0, it follows that x2 > M 2 > 0. So,
ε
1 1
< 2 . We obtain
x2 M
1 1 1 1
2
−0 = 2 < 2 < 2 = ε.
x +1 x +1 x M
1
We conclude that lim = 0.
x→∞ x2 + 1
1
Example 4.3.5 Prove that lim = 0.
x→−∞ x
1
Proof. Let ε > 0. Choose M = − < 0. If x < M < 0, it implies −x > −M > 0. We obtain
ε
1 1 1
−0 = < = ε.
x −x −M
1
We conclude that lim = 0.
x→−∞ x
x
Example 4.3.6 Prove that lim = 1.
x→−∞ x + 1
1 1
Proof. Let ε > 0. Choose M = −1 − . Then M + 1 = − < 0. If x < M , it implies
ε ε
1 1
1 + x < 1 + M < 0. So, 0 < − <− . We obtain
x+1 M +1
x 1 1 1
−1 = = < = ε.
x+1 |x + 1| −(x + 1) −(M + 1)
x
We conclude that lim = 1.
x→−∞ x + 1
100 CHAPTER 4. LIMIT OF FUNCTIONS
1
Proof. Let M > 0. Choose δ = > 0. If 0 < |x| < δ, it follows
M
1 1
> = M.
|x| δ
1
Thus, lim = +∞.
x→0 |x|
x
Example 4.3.9 Prove that lim+ = −∞.
x→1 1−x
1 1 1 1 1
Proof. Let M < 0. Choose δ = − > 0. If 0 < x − 1 < δ, it follows < . So, <− .
M δ x−1 1−x δ
We obatin
x 1 1 1
= −1 + <0+ < − = M.
1−x 1−x 1−x δ
x
Thus, lim+ = −∞.
x→1 1−x
4.3. INFINITE LIMIT 101
Exercises 4.3
1. Use definitons to prove that lim+ f (x) exists and equal to L in each of the following cases.
x→a
1
1.1 f (x) = , a = 3, and L = +∞.
x−3
1
1.2 f (x) = − , a = 0, and L = −∞.
x
2. Use definitons to prove that lim− f (x) exists and equal to L in each of the following cases.
x→a
x
2.1 f (x) = , a = 2, and L = −∞.
−4
x2
1
2.2 f (x) = , a = 1, and L = +∞.
1 − x2
2x + 1 x
3.1 lim =2 3.4 lim = +∞
x→∞ x + 1 x→2 |x − 2|
1−x 1 x+1
3.2 lim =− 3.5 lim+ = +∞
x→−∞ 2x + 1 2 x→2 x − 2
2
2x + 1 x+1
3.3 lim = −2 3.6 lim− = −∞
x→∞ 1 − x2 x→2 x − 2
x3 − 1
4.3 lim 2 4.6 lim x2 sin x
x→−∞ x + 2 x→−∞
sin(x + 3) − sin 3
5. Prove that converges to 0 as x → ∞.
x
6.2 If f (x) ≤ g(x) ≤ h(x) and L = lim f (x) = lim h(x), then g(x) → L as x → ∞.
x→∞ x→∞
102 CHAPTER 4. LIMIT OF FUNCTIONS
for every a ∈ R.
P (x)
7.3 Suppose that P is a polynomial and P (a) > 0. Prove that → ∞ as x → a+ ,
x−a
P (x)
→ −∞ as x → a− , but
x−a
P (x)
lim
x→a x−a
does not exist.
lim f (n + 1) − f (n) = L,
n→∞
f (n)
prove that lim exists and equals L.
n→∞ n
Chapter 5
Continuity on R
5.1 Continuity
Thus, f is continuous at x = 1.
ε
|f (x) − f (2)| = |x2 − 4| = |x + 2||x − 2| < (|x| + 2)δ < (3 + 2) = ε.
5
Thus, f is continuous at x = 2.
104 CHAPTER 5. CONTINUITY ON R
√
Example 5.1.4 Let f (x) = x where x ∈ (0, ∞). Prove that f is continuous at 1.
Thus, f is continuous at x = 2.
Example 5.1.5 Let f (x) = 3 − x2 where x ∈ [−1, 2] ∪ {3}. Prove that f is continuous at x = 3
Proof. Let ε > 0. Choose δ = 0.5. Let x ∈ [−1, 2] ∪ {3} such that |x − 3| < δ = 0.5. It follows
x = 3. We obtain
Thus, f is continuous at x = 3.
is discontinuous at 0.
x |x|
1= = = |f (x)| < 1
x x
Theorem 5.1.7 Let I be an open interval that contain a point a and f : I → R. Then
( )
c a d
a−c
is identical to
a+1 = a+1 = 5
Hence, a = 4.
106 CHAPTER 5. CONTINUITY ON R
1. f is continuous at a ∈ E.
n
xn = .
n+1
Next, we will use Theorem 5.1.10 together Theorems of limit for addition, mutiplication, scalar
multiplication and quotient in order to proof Theorem 5.1.12.
f + g, f g and αf
CONTINUITY OF COMPOSITION.
Definition 5.1.13 Suppose that A and B are subsets of R and that f : A → R and g : B → R.
If {f (x) : x ∈ A} ⊆ B, then the composition of g with f is the function
R R
A
B
f g
x f (x) g(f (x))
g◦f
Theorem 5.1.14 Suppose that A and B are subsets of R and that f : A → R and g : B → R with
{f (x) : x ∈ A} ⊆ B. If f is continuous at a ∈ A and g is continuous at f (a) ∈ B, then
g ◦ f is continuous at a ∈ A
and moreover,
( )
lim (g ◦ f )(x) = g lim f (x) .
x→a x→a
For each x ∈ A such that |x − a| < δ2 , it implies |f (x) − f (a)| < δ1 . Set y = f (x).
We obtain by (5.2) that |g(f (x)) − g(f (a))| < ε. We conclude that g ◦ f is continuous at a ∈ A.
108 CHAPTER 5. CONTINUITY ON R
√
Example 5.1.15 Show that lim 2x − 1 exists and equals to 1.
x→1
√
Solution. Let g(x) = x and f (x) = 2x − 1. Then f is continous at 1 and g is continuous at
f (1) = 1. By Theorem 5.1.14,
( ) ( )
lim (g ◦ f )(x) = g lim f (x) = g lim (2x − 1) = g(1) = 1.
x→1 x→1 x→1
CONTINUITY ON A SET.
We obtain
Theorem 5.1.18 (Continuity of linear function) Let m and c be constants and let
f (x) = mx + c where x ∈ R.
ε
Proof. Let a ∈ R and ε > 0. Choose δ = > 0. Let x ∈ R such that |x − a| < δ. We obtain
|m| + 1
Solution. Let f (x) = x2 and g(x) = 3x + 1. By Example 5.1.17 and Theorem 5.1.18, f and g are
continuous on R. We conclude by Theorem 5.1.14 that
Solution. We see that f is a linear function on (−1, ∞) ∪ (−1, ∞). By Continuity of Linear
function, f is continuous on (−1, ∞) ∪ (−1, ∞). From
2a + 1 = 2a + 1 = 2 + a.
Hence, a = 1.
110 CHAPTER 5. CONTINUITY ON R
Exercises 5.1
4. Use limit theorem to show that the following function are continuous on [0, 1].
√
4.1 f (x) = 3x2 + 1 4.3 f (x) = 2−x
1−x 1
4.2 f (x) = 4.4 f (x) =
1+x x2 +x−6
ax + 3 if x ≤ 1
5. Find a and b such that the function f (x) = x + b if 1 < x ≤ 2 is continuous on R.
2ax − 2 if x > 2
7. Show that there exist nowhere continuous functions f and g whose sum f + g is continuous
on R. Show that the same is ture for product of functions.
5.1. CONTINUITY 111
8. Let
cos( 1 )
x
if x ̸= 0
f (x) =
1 if x = 0
is continuous on (−∞, 0) and (0, ∞), discontinuous at 0, and neither f (0+ ) nor f (0− ) exists.
8.2 Suppose that g : [0, π2 ] → R is continuous on (0, π2 ) and that there is a positive constant
C > 0 such that
√
|g(x)| ≤ C x for all x ∈ (0, π2 ),
9.2 Make and prove an analogous atstement for the product f g. Show by example that
hypothesis about f added cannot be dropped.
12.4 Prove that f is continuous at 0, then there is an m ∈ R such that f (x) = mx for all
x ∈ R.
ln(x + 1) 1
13. Assume that lim = 1 and f (x) = ex is continuous on R. Show that lim (1+x) x = e.
n→0 x x→0
112 CHAPTER 5. CONTINUITY ON R
So, f is bounded by 1 on R.
Next, let f : I → R be a function. We define
Theorem 5.2.2 (Extreme Value Theorem (EVT)) If I is a closed, bounded interval and
f : I → R is continuous on I, then f is bounded on I. Moreover, if
Proof. Suppose that f is not bounded in I. Then there exist xn ∈ I such that
Since I is bounded, we know by the Bolzano-Weierstrass Theorem that {xn } has a convergent
subsequence, say xnk → a as k → ∞. Since I is closed, we also know by the Comparison Theorem
that a ∈ I and f (a) ∈ R. By (5.4), we obtain
1
f (x) ≤ M − .
C
We obtain
1
M = sup f (x) ≤ M − < M.
x∈I C
It is imposible. Thus, there is an xM ∈ I such that f (xM ) = M . A similar argument proves that
there is an xm ∈ I such that f (xm ) = m.
f (b) y = f (x)
y0
f (a)
X
a x0 b
Since a ∈ E and E ⊆ [a, b], E is a nonempty bounded subset of R. Thus, by the Completeness
Axiom, x0 = sup E is a finite real number. Since y0 is equals neither f (a) nor f (b), x0 cannot
equal to a or b. Hence, x0 ∈ (a, b).
It remains to show that f (x0 ) = y0 . By Theorem 2.2.5, there is a sequence xn ∈ E such that
xn → sup E = x0 as n → ∞.
Since f is continuous and the definition of E, by the Comparison Theorem and Theorem 5.1.10
we obtain
f (x0 ) = lim f (xn ) ≤ y0 .
n→∞
Finally, we will prove that f (x0 ) = y0 , suppose to the contrary that f (x0 ) < y0 . Set
Then g is continuous and g(x0 ) > 0. Hence, by Lemma 5.2.3, we can choose positive numbers ε
and δ such that
For any x, it satisfies x0 < x < x0 + δ also satisfies y0 − f (x) = g(x) > 0 or f (x) < y0 which
contradics the fact that x0 = sup E.
5.2. INTERMEDIATE VALUE THEOREM 115
1. If f (a) > 0 and f (b) < 0, then there is an c ∈ (a, b) such that f (c) = 0.
2. If f (a) < 0 and f (b) > 0, then there is an c ∈ (a, b) such that f (c) = 0.
c2 − c − 1 = f (c) = 0.
Example 5.2.7 Prove that ln x = 3 − 2x has at least one real root and find the approximate root
to be the midpont of an interval [a, b] of length 0.01 that contain a root.
Since f is continuous on (1.34, 1.35), we obatin by Corollary 5.2.5 that there is an c ∈ (1.34, 1.35)
such that
ln c + 2c − 3 = f (c) = 0.
Exercises 5.2
For these exercise, assume that sin x, cos x and ex are continuous on R and ln x is continuous on
R+ .
1. For each of the following, prove that there is at least one x ∈ R that satisfies the given
equation.
1.1 x3 + x = 3 1.6 ex = x2
1.2 x3 + 2 = 2x 1.7 x ln x = 1
2. Prove that the follwing equations have at least one real root and find the approximate root
to be the midpont of an interval [a, b] of length ℓ that contain a root.
4. If f : R → R is continuous and
Solution. Let ε > 0. Choose δ = ε. Let x, a ∈ (0, 1) such that |x − a| < δ. We obtain
Proof. Suppose to the contrary that f is continuous but not uniformly continuos on I.
Then there is an ε0 > 0 such that
Then sequence {xn } and {yn } are bounded. By The Bolzano-Weierstrass Theorem, {xn } has a
subsequence, say xnk , that converges, as k → ∞, to some x ∈ I. Similarly, {yn } has a subsequence,
say ynj , that converges, as j → ∞, to some y ∈ I. Since xnj → x as j → ∞ and f is continuous,
it follows by the Comparison Theorem from (5.6) that
1
So, f (x) ̸= f (y). But |xn − yn | < for all n ∈ R, so Theorem 1.3.10 implies that x = y. Thus,
n
f (x) = f (y), a contradiction.
5.3. UNIFORM CONTINUITY 119
For each n, m ≥ N such that |xn − xm | < δ it satisfies (5.7) that we have
Exercises 5.3
1. Use Definition to prove that each of the following functions is uniformly continuous on (0, 1).
2. Prove that each of the following functions is uniformly continuous on (0, 1).
1
3. Prove that f (x) = is uniformly continuous on R.
x2 +1
4. Find all real α such that xα sin( x1 ) is uniformly continuous on the open interval (0, 1).
8.3 Show that there exist functions f, g uniformly continuous on R such that f g is not
uniformly continuous on R.
Differentiability on R
You may recall that the graph of y = f (x) has a tangent line at the point (a, f (a)) if and
only if f has a derivative at a, in which case the slope of that tangent line is f ′ (a). Suppose that
f is differentiable at a. A secant line of the graph y = f (x) is a line passing through at least
two points on the graph, an a chord is a line segment that runs from one point on the graph to
another.
Y
y = f (x)
Chord
Tangent
a x2 x1 X
122 CHAPTER 6. DIFFERENTIABILITY ON R
Let x = a + h and observe that the slope of the chord (chord function : F (x)) passing through
the points (x, f (x)) and (a, f (a)) is given by
f (x) − f (a)
F (x) := , x ̸= a.
x−a
Now, since x = a + h, f ′ (a) becomes
f (x) − f (a)
f ′ (a) = lim .
x→a x−a
Example 6.1.2 Let f (x) = x2 where x ∈ R. Find f ′ (1)
Solution. We consider
Solution. Consider
( )
f (x) − f (0) x2 cos( x1 ) 1
lim = lim = lim x cos = 0.
x→0 x−0 x→0 x x→0 x
By Example 4.1.18, f ′ (0) = 0. Thus, f is differentiable at the origin.
Solution. We consider
( )
f (x) − f (0) x cos( x1 ) 1
lim = lim = lim cos .
x→0 x−0 x→0 x x→0 x
By Example 4.1.12, the limit does not exist. Thus, f is not differentiable at the origin.
6.1. THE DERIVATIVE 123
f (a + h) − f (a) − T (h)
lim = 0.
h→0 h
f (a + h) − f (a) − T (h)
Conversely, assume that lim = 0. Then
h→0 h
f (a + h) − f (a) f (a + h) − f (a) − mh
lim − m = lim
h→0 h h→0 h
f (a + h) − f (a) − T (h)
= lim = 0.
h→0 h
f (x) − f (a)
f (x) − f (a) = · (x − a).
x−a
f (x) − f (a)
lim f (x) − f (a) = lim · (x − a) = f ′ (a) · 0 = 0
x→a x→a x−a
Example 6.1.7 Show that f (x) = |x| is continuous at 0 but not differentiable there.
does not exist by Example 4.2.3. Thus, f is not differentiable at 0 but it easy to prove that f
continuous at 0.
DIFFERENTIABLE ON INTERVAL.
f ′ (x) = nxn−1 .
Theorem 6.1.11 Every constant function is differentiable on R and its value equals to zero.
f (x + h) − f (x) c−c
f ′ (x) = lim = lim = lim 0 = 0
h→0 h h→0 h h→0
Exercises 6.1
1. For each of the following real functions, use definition directly to prove that f ′ (a) exists.
3. Let I be an open interval that contains 0 and f : I → R. If there exists an α > 1 such that
Theorem 6.2.1 (Additive Rule) Let f and g be real functions. If f and g are differentiable at
a, then f + g is differentiable at a. In fact,
(f + g)(a + h) − (f + g)(a)
(f + g)′ (a) = lim
h→0 h
[f (a + h) − f (a)] + [g(a + h) − g(a)]
= lim
h→0 h
f (a + h) − f (a) g(a + h) − g(a)
= lim + lim
h→0 h h→0 h
= f ′ (a) + g ′ (a)
αf (a + h) − αf (a)
(αf (a))′ = lim
h→0 h
f (a + h) − f (a)
= α · lim
h→0 h
= αf ′ (a).
Theorem 6.2.3 (Product Rule) Let f and g be real functions. If f and g are differentiable at
a, then f g is differentiable at a. In fact,
Theorem 6.2.4 (Quotient Rule) Let f and g be real functions. If f and g are differentiable at
f
a, then is differentiable at a when g(a) ̸= 0. In fact,
g
( )′
f g(a)f ′ (a) − f (a)g ′ (a)
(a) = .
g [g(a)]2
Example 6.2.5 Let f and g be differentiable at 1 with f (1) = 1, g(1) = 2 and f ′ (1) = 3, g ′ (1) = 4.
Evaluate the following derivatives.
Theorem 6.2.6 (Chain Rule) Let f and g be real functions. If f is differentiable at a and g is
differentiable at f (a), then g ◦ f is differentiable at a with
f (x) − f (a)
f (x) = · (x − a) + f (a), x ̸= a
x−a
g(y) − g(f (a))
g(y) = · (y − f (a)) + g(f (a)), y ̸= f (a) (6.1)
y − f (a)
Example 6.2.7 Let f and g be differentiable on R with f (0) = 1, g(0) = −1 and f ′ (0) = 2,
g ′ (0) = −2 ,f ′ (−1) = 3, g ′ (1) = 4. Evaluate each of the following derivatives.
√ x
Example 6.2.8 Let f (x) = x2 + 1. Use the Chain Rule to show that f ′ (x) = √ .
2
x +1
√
Solution. Let g(x) = x and h(x) = x2 + 1. We have
1
g ′ (x) = √ and h′ (x) = 2x.
2 x
By Chain Rule,
Exercises 6.2
1. For each of the following functions, find all x for which f ′ (x) exists and find a formula for f ′ .
2. Let f and g be differentiable at 2 and 3 with f ′ (2) = a, f ′ (3) = b, g ′ (2) = c and g ′ (3) = d, If
f (2) = 1, f (3) = 2 ,g(2) = 3 and g(3) = 4. Evaluate each of the following derivatives.
( )′
2.1 (f g)′ (2) 2.2 f
(3) 2.3 (g ◦ f )′ (3) 2.4 (f ◦ g)′ (2)
g
Lemma 6.3.1 (Rolle’s Theorem) Suppose that a, b ∈ R with a ̸= b. If f is continuous on [a, b],
differentiable on (a, b), and if f (a) = f (b), then f ′ (c) = 0 for some c ∈ (a, b).
Y
f ′ (c) = 0
y = f (x)
f (a) = f (b)
a c X
b
Proof. Let a ̸= b such that f is continuous on [a, b] and differentiable on (a, b).
Assume that f (a) = f (b). By EVT, f has a finite maximum M and a finite minimum m on [a, b].
Case M = m. Then f is a constant function. Thus, f ′ (x) = 0 for all x ∈ (a, b).
Case M ̸= m. Since f (a) = f (b), there is a c ∈ (a, b) such that f (c) = M . We have
f (c + h) − f (c)
f ′ (c) = lim+ ≤ 0,
h→0 h
f (c + h) − f (c)
f ′ (c) = lim− ≥ 0.
h→0 h
Y
Tangent
Chord
y = f (x)
a c X
b
Proof. Let a ̸= b such that f is continuous on [a, b] and differentiable on (a, b). We set
We obtain
Solution. Let a > 0 and define f (x) = sin x where x ∈ [0, a] . Then f is continuous on [0, a] and
f (x) is differentiable and f ′ (x) = cos x for every x ∈ (0, a).
By the MVT, there is a c ∈ (0, a) such that
sin a − 0 cos c · a
sin a = a cos c
From cos c ≤ 1 and a > 0, a cos c ≤ a, it implies that sin a < a. Therefore,
Solution. Let a > 0 and define f (x) = ex − x − 1 where x ∈ [0, a] . Then f is continuous on [0, a]
and f (x) is differentiable and f ′ (x) = ex − 1 for every x ∈ (0, a).
By the MVT, there is a c ∈ (0, a) such that
ea − a − 1 = (ec − 1)a
Example 6.3.5 (Bernoulli’s Inequality) Let 0 < α ≤ 1 and δ ≥ −1. Prove that
(1 + δ)α ≤ 1 + αδ.
Proof. Let 0 < α ≤ 1 and δ ≥ −1. Define f (x) = xα where x ∈ R. Then f is continuous on R and
f (x) is differentiable and
1 − (1 + δ)α = −δαcα−1
(1 + δ)α − 1 = δαcα−1
Since 0 < α ≤ 1, −1 < α − 1 ≤ 0. From 0 ≤ 1 + δ < c < 1, it implies that cα−1 ≥ c0 = 1. Since
δ ≤ 0 and α > 0, δα ≤ 0 which leads to δαcα−1 ≤ αδ. Thus,
(1 + δ)α ≤ 1 + αδ.
f (1 + δ) − f (1) = f ′ (c)[(1 + δ) − 1]
(1 + δ)α − 1 = δαcα−1
Since 0 < α ≤ 1, −1 < α − 1 ≤ 0. From c > 1, it implies that cα−1 ≤ c0 = 1. Since δ > 0 and
α > 0, δα > 0 which leads to δαcα−1 ≤ αδ. Thus,
(1 + δ)α ≤ 1 + αδ.
Proof. Let a ̸= b such that f and g are continuous on [a, b] and differentiable on (a, b). We set
We obtain
= h(b).
Theorem 6.3.7 (L’Hôspital’s Rule) Let a be an extended real number and I be an open interval
that either contains a or has a as an endpoint. Suppose that f and g are differentiable on I\{a},
and g(x) ̸= 0 ̸= g ′ (x) for all x ∈ I\{a}. Suppose further that
is either 0 or ∞. If
f ′ (x)
B := lim
x→a g ′ (x)
f (x) f ′ (x)
lim = lim ′ .
x→a g(x) x→a g (x)
f (x)
lim = B.
x→a g(x)
Let xk ∈ I\{a} such that xk → a as k → ∞. Note that if g ′ is never zero on I\{a}. By the MVT,
for x, y < a or x, y > a there is a c ∈ (x, y) such that
1
Given (ln x)′ = for x > 0 and (ex )′ = ex for all x ∈ R.
x
x
Example 6.3.8 Use L’Hôspital’s Rule to prove that lim = 1.
x→0 ex −1
Solution. We see that
lim x = 0 = lim ex − 1.
x→0 x→0
x (x)′ 1
lim = lim = lim x = 1.
x→0 e − 1
x x→0 (e − 1)
x ′ x→0 e
ln x (ln x)′
lim+ x ln x = lim+ = lim
x→0 x→0 x−1 x→0+ (x−1 )′
x−1
= lim+ = lim+ (−x) = 0.
x→0 −x−2 x→0
(ln(ln x))′
ln L = ln lim− (ln x)1−x = lim− ln(ln x)1−x = lim− (1 − x) ln(ln x) = lim− 1 ′
x→1 x→1 x→1 x→1 ( 1−x )
1
ln x x
·1 (1 − x)2
= lim− 1 = lim−
x→1
(1−x)2
x→1 x ln x
Exercises 6.3
1. Use the Mean Value Theorem to prove that each of the following inequalities.
√ x−1
1.1 2x + 1 < 1 + x for all x > 0 1.6 ≤ ln x for all x > 1
x
1.2 ln x ≤ x − 1 for all x > 1 √
1.7 x ≥ x for all x ∈ [0, 1]
1.3 7(x − 1) < ex √
for all x > 2 1.8 x≤x for all x > 1
1.4 cos x − 1 ≤ x for all x > 0 1.9 sin2 x ≤ 2|x| for all x ∈ R
2
x +1 √
1.5 ln x + 1 ≤ for all x > 1 1.10 ln x ≤ x for all x > 1
2
(1 + δ)α ≤ 1 + αδ.
( )x
sin(3x) 3.4 lim+ xx 3.7 lim− 1 + e−x
3.1 lim x→0 x→0
x→0 x
cos x − ex ln x 1
3.2 lim+ 3.5 lim 3.8 lim (1 + x) x
x→0 ln(1 + x2 ) x→1 sin(πx) x→0
( x ) 12 ( π) 1
3.6 lim x arctan x − 3.9 lim x(e x − 1)
x
3.3 lim
x→0 sin x x→∞ 2 x→∞
5.1 If f ′ (x) = 0 for all x ∈ R, prove that f (x) = f (0) for all x ∈ R
5.2 If f (0) = 1 and |f ′ (x)| ≤ 1 for all x ∈ R, prove that |f (x)| ≤ |x| + 1 for all x ∈ R
5.3 If ′ (x) ≥ 0 for all x ∈ R, prove that a < b imply that f (a) < f (b)
6.3. MEAN VALUE THEOREM 141
6. Let f be differentiable on a nonempty, open interval (a, b) with f ′ bounded on (a, b). Prove
that f is uniformly continuous on (a, b).
7. Let f be differentiable on (a, b), continuous on [a, b], with f (a) = f (b) = 0. Prove that if
f ′ (c) > 0 for some c ∈ (a, b), then there exist x1 , x2 ∈ (a, b) such that f ′ (x1 ) > 0 > f ′ (x2 ).
8. Let f be twice differentiable on (a, b) and let there be points x1 < x2 < x3 in (a, b) such that
f (x1 ) > f (x2 ) and f (x3 ) > f (x2 ). Prove that there is a point c ∈ (a, b) such that f ′′ (c) > 0.
9. Let f be differentiable on (0, ∞). If L = lim f ′ (x) and lim f (n) both exist and are finite,
x→∞ n→∞
prove that L = 0.
10. Prove L’Hôspital’s Rule for the case B = ±∞ by first proving that
g(x) f (x)
→ 0 when → ±∞, as x → a.
f (x) g(x)
( )n
1
11. Prove that the sequence 1 + is increasing, as n → ∞, and its limit e satisfies 2 < e ≤ 3
n
and ln e = 1.
142 CHAPTER 6. DIFFERENTIABILITY ON R
Example 6.4.2 Show that f (x) = x2 is strictly monotone on [0, 1] and on [−1, 0] but not monotone
on [−1, 1].
Solution.
If 0 ≤ x < y ≤ 1, then x2 < y 2 , i.e., f (x) < f (y). Thus, f is strictly increasing on [0, 1].
If −1 ≤ x < y ≤ 0, then x2 > y 2 , i.e., f (x) > f (y). Thus, f is strictly decreasing on [−1, 0].
We conclude that f is strictly monotone on [0, 1] and on [−1, 0].
Since f is increasing and decreasing on [−1, 1], f is not monotone on [−1, 1].
6.4. MONOTONE FUNCTION 143
If f ′ (x) > 0 for all x ∈ (a, b), f ′ (c) > 0. It follows that f (y) > f (x). So, f is increasing on (a, b).
If f ′ (x) < 0 for all x ∈ (a, b), f ′ (c) < 0. It follows that f (y) < f (x). So, f is decreasing on (a, b).
Let x ∈ [a, b]. By the MVT, there is a c ∈ (a, x) such that
So, f (x) = f (a) for all x ∈ [a, b]. We conclude that f is constant on [a, b].
Example 6.4.4 Find each intervals of f (x) = x2 − 4x + 3 that increasing and decreasing.
Theorem 6.4.5 If f is 1-1 and continuous on an interval I, then f is strictly monotone on I and
f −1 is continuous and strictly monotone on f (I) := {f (x) : x ∈ I}.
Proof. Assume that f is 1-1 and continuous on an interval I. Let a, b ∈ I such that
Suppose that f is not strictly monotone on I. Then there exist points a, b, c ∈ I such that a < c < b
but f (c) does not lie between f (a) and f (b). It follows that either f (a) lie between f (b) and f (c)
or f (b) lie between f (a) and f (c). Hence by the IVT, there is an x1 ∈ (a, b) such that
Set y = f −1 (x). Then f (x0 ) < f (x) < f (x0 + ε0 ). Since f is strictly increasing on I, it implies
x0 < x < x0 + ε0 , i.e., 0 < x − x0 < ε0 . We conclude that
So, f −1 (y0+ ) = f −1 (y0 ). A similar argument show that if y0 is not a left endpoint of f (I),
f −1 (y0− ) = f −1 (y0 ). Hence, f −1 is continuous on f (I).
6.4. MONOTONE FUNCTION 145
Theorem 6.4.6 (Inverse Function Theorem (IFT)) Let f be 1-1 and continuous on an open
interval I. If a ∈ f (I) and if f ′ (f −1 (a)) exists and is nonzero, then f −1 is differentiable at a and
1
(f −1 )′ (a) = .
f ′ (f −1 (a))
Proof. Let f be 1-1 and continuous on an open interval I. By Theorem 6.4.5, f is strictly monotone,
say strictly increasing on I and f −1 exists, is continuous and strictly increasing on f (I).
Assume that a ∈ f (I) and f ′ (f −1 (a)) exists and is nonzero. Set x0 = f −1 (a) ∈ I and I is open,
we can choose c, d ∈ R such that x0 ∈ (c, d) ⊂ I. Then a = f (x0 ) ∈ (f (c), f (d)) ⊂ f (I).
We can choose h ̸= 0 so small that a + h ∈ f (I). i.e., f −1 (a + h) exists. Set x = f −1 (a + h) and
observe that f (x) − f (x0 ) = a + h − a = h. Since f −1 is continuous, x → x0 if and only if h → 0.
Therefore,
f −1 (a + h) − f −1 (a) x − x0 1 1
(f −1 )′ (a) = lim = lim = ′ = ′ −1 .
h→0 h x→x0 f (x) − f (x0 ) f (x0 ) f (f (a))
Example 6.4.7 Use the Inverse Function Theorem to find derivative of f (x) = arcsin x
1
f ′ (x) = (arcsin x)′ = (g −1 )′ (x) =
g ′ (g −1 (x))
1
=
g ′ (arcsin x)
1
=
cos(arcsin x)
1
=√
1 − sin2 (arcsin x)
1
=√ .
1 − x2
146 CHAPTER 6. DIFFERENTIABILITY ON R
2. Use the result from 1 and the IFT to explain that f −1 differentiable on R.
3. Compute (f −1 )′ (2 + ln 2).
Solution.
1. Proof. Let x, y ∈ R and x ̸= y. WLOG x > y. Then x − y > 0 and ex > ey . We obtain
ey − ex < 0 < x − y
y + ey < x + ex
2. Since f is 1-1, f −1 exists. It is clear that f is continous on R. By the IFT, we conclude that
f −1 differentiable on R.
1 1 1 1
(f −1 )′ (2 + ln 2) = = = = .
f ′ (f −1 (2 + ln 2)) f ′ (ln 2) 1+2 3
6.4. MONOTONE FUNCTION 147
Exercises 6.4
1. Find each intervals of the following functions that increasing and decreasing.
3. Find all a ∈ R such that ax2 + 3x + 5 is strictly increasing on the interval (1, 2).
√
4. Find where f (x) = 2|x − 1| + 5 x2 + 9 is strictly increasing and where f (x) is strictly
decreasing.
5. Let f and g be 1-1 and continuous on R. If f (0) = 2, g(1) = 2, f ′ (0) = π, and g ′ (1) = e,
compute the following derivatives.
6. Let f (x) = x2 ex , x ∈ R.
2
7.2 Use the result from 7.1 and the IFT to explain that f differentiable on R.
√ 1
8.3 ( x)′ = √ where x ∈ (0, ∞)
2 x
9. Use the IFT to find derivative of invrese function f (x) = ex − e−x where x ∈ R.
10. Suppose that f ′ exists and continuous on a nonempty, open interval (a, b) with f ′ (x) ̸= 0 for
all x ∈ (a, b).
10.1 Prove that f is 1-1 on (a, b) and takes (a, b) onto some open interval (c, d)
10.3 Use the function f (x) = x3 , show that 7.2 is false if the assumption f ′ (x) ̸= 0 fails to
hold for some x ∈ (c, d)
11. Let [a, b] be a closed, bounded interval. Find all functions f that satisfy the following
conditions for some fixed α > 0 : f is continuous and 1-1 on [a, b],
12. Let f be differentiable at every point in a closed, bounded interval [a, b]. Prove that if f ′ is
increasing on (a, b), then f ′ is continuous on (a, b).
Integrability on R
PARTITION.
1. A partition of the interval [a, b] is a set of points P = {x0 , x1 , , ..., xn } such that
Example 7.1.2 Give example of partition and refinement of the interval [0, 1].
3. Find a norm of Pn .
0 ≤ j · 2m−n ≤ 2n · 2m−n = 2m .
We obtain
j · 2m j · 2m−n
x= = ∈ Pm .
2n · 2m 2m
Thus, Pm is finer than Pn when m > n. We final have
j j−1 1
∥Pn ∥ = max − = .
1≤j≤n 2n 2n 2n
7.1. RIEMANN INTEGRAL 151
Definition 7.1.5 Let a, b ∈ R with a < b, let P = {x0 , x1 , , ..., xn } be a partition of the interval
[a, b], and suppose that f : [a, b] → R is bounded.
where
Mj (f ) := sup f (x).
x∈[xj−1 ,xj ]
where
mj (f ) := inf f (x).
x∈[xj−1 ,xj ]
Y
y = f (x)
X
a = x0 x1 x2 x3 xk−1 xk xn−1 xn = b
Y
y = f (x)
X
a = x0 x1 x2 x3 xk−1 xk xn−1 xn = b
152 CHAPTER 7. INTEGRABILITY ON R
Example 7.1.6 Let f (x) = x2 + 1 where x ∈ [0, 1]. Find L(f, P ) and U (f, P )
{ }
1 1 3
1. P = 0, , , , 1
4 2 4
Y f (x) = x2 + 1 Y f (x) = x2 + 1
2 2
1 1
X X
0 1 1 3 1 0 1 1 3 1
4 2 4 4 2 4
( ) ( ) ( )
1 1 1 1 1 1 3
L(P, f ) = f (0) + f + f + f
4 4 4 4 2 4 4
( )
1 17 5 25 79
= 1+ + + =
4 16 4 16 64
( ) ( ) ( )
1 1 1 1 1 3 1
U (P, f ) = f + f + f + f (1)
4 4 4 2 4 4 4
( )
1 17 5 25 47
= + + +2 =
4 16 4 16 32
Y f (x) = x2 + 1 Y f (x) = x2 + 1
2 2
1 1
X X
0 0.2 0.5 0.6 0.8 1 0 0.2 0.5 0.6 0.8 1
L(P, f ) = 0.2f (0) + 0.3f (0.2) + 0.1f (0.5) + 0.2f (0.6) + 0.2f (0.8)
= 1.237
U (P, f ) = 0.2f (0.2) + 0.3f (0.5) + 0.1f (0.6) + 0.2f (0.8) + 0.2f (1)
= 1.447
7.1. RIEMANN INTEGRAL 153
Example 7.1.7 Let f (x) = x2 + 1 where x ∈ [0, 1]. Find L(Pn , f ) and U (Pn , f ) for n ∈ N if
{ }
j
Pn = : j = 0, 1, ..., n .
n
k 1
Solution. Let xk = and ∆xk = xk − xk−1 = for each k = 0, 1, 2, ..., n.
n n
2 f (x) = x2 + 1
1
... ...
X
0 1 2 3 k−1 k n−1 1
n n n n n n
Thus, we obtain
∑ ∑n [ ]
1 ∑ 1∑
n n n
1 1
L(Pn , f ) = mk ∆xk = (k − 1) + 1
2
= 3 (k − 1) +
2
1
k=1 k=1
n2 n n k=1 n k=1
1 2 1
= [0 + 1 2
+ 22
+ · · · + (n − 1) 2
] + ·n
n3 n
1 (n − 1)(n − 1 + 1)(2(n − 1) + 1)
= 3· +1
n 6
(n − 1)(n)(2n − 1) (n − 1)(2n − 1)
= 3
+1= +1
6n 6n2
and
∑ ∑n [ ]
1 ∑ 2 1∑
n n n
1 1
U (Pn , f ) = Mk ∆xk = ·k +1
2
= 3 k + 1
k=1 k=1
n2 n n k=1 n k=1
1 n(n + 1)(2n + 1) 1
= · + ·n
n3 6 n
(n + 1)(2n + 1)
= + 1.
6n2
154 CHAPTER 7. INTEGRABILITY ON R
Theorem 7.1.8 L(f, P ) ≤ U (f, P ) for all partition P and all bounded function f .
Proof. Let P = {x0 , x1 , ..., xn } be a partition and f be bounded on [a, b]. Then
It follows that
∑
n ∑
n
L(f, P ) = mj (f )∆xj ≤ Mj (f )∆xj = U (f, P ).
j=1 j=1
∑
n
[g(k + 1) − g(k)] = g(n + 1) − g(m)
k=m
for all n ≥ m in N.
Proof. Fix m ∈ N. We will prove by induction on n. The Sum Telescopes is obvious for n = 1.
Assume that the Sum Telescopes is true for some n ∈ N. By inductive hypothesis,
∑
n+1 ∑
n
[g(k + 1) − g(k)] = [g(k + 1) − g(k)] + [g(n + 2) − g(n + 1)]
k=m k=m
= g(n + 2) − g(m).
The Sum Telescopes is true for some n+1. We conclude that by induction that the Sum Telescopes
holds for n ∈ N.
7.1. RIEMANN INTEGRAL 155
Proof. Let f (x) = α is constant on [a, b] and let P = {x0 , x1 , x2 , ..., xn } be a partition of [a, b] such
that x0 = a and xn = b.
Y
α
... ...
X
a = x0 x1 x2 x3 xk−1 xk xn−1 xn = b
Suppose c ∈
/ P . Then there is an xk such that
Consider
∑
k−1 ∑
n
U (f, P ) = Mj (f )∆xj + sup f (x) · (xk − xk−1 ) + Mj (f )∆xj
j=1 x∈[xk−1 ,xk ] j=K+1
∑
k−1 ∑
n
U (f, Q) = Mj (f )∆xj + sup f (x) · (c − xk−1 ) + sup f (x) · (xk − c) + Mj (f )∆xj
j=1 x∈[xk−1 ,c] x∈[c,xk ] j=k+1
We obtain
U (f, P ) − U (f, Q) = sup f (x) · (xk − xk−1 ) − sup f (x) · (c − xk−1 ) − sup f (x) · (xk − c)
x∈[xk−1 ,xk ] x∈[xk−1 ,c] x∈[c,xk ]
Thus, U (f, P ) ≥ U (f, Q). A similar argument show that L(f, P ) ≤ L(f, Q).
Proof. Assume that P and Q are any partitions of [a, b]. Then
P ⊆P ∪Q and Q ⊆ P ∪ Q.
RIEMANN INTEGRABLE.
Theorem 7.1.14 Suppose that a, b ∈ R with a < b. If f is continuous on the interval [a, b], then
f is integrable on [a, b].
Proof. Let a, b ∈ R with a < b. Assume that f is continuous on the interval [a, b].
It follows that f is bounded on [a, b] by the EVT. Theorem 5.3.6 implies that f is uniformly
continuous on the interval [a, b]. Let ε > 0. There is a δ > 0 such that
ε
|x − y| < δ and x, y ∈ [a, b] imply |f (x) − f (y)| < . (7.1)
b−a
Since ∥P ∥ < δ, we have |xM − xm | ≤ |xj − xj−1 | < δ. Then xm , xM satisfy (7.1), it implies that
ε
|Mj (f ) − mj (f )| = |f (xM ) − f (xm )| < .
b−a
Y
1
X
0 1
2
− ε
4
1
2
1
2
+ ε
4
1
We obtain
[( ) ] [( ) ( )] [ ( )]
1 ε 1 ε 1 ε 1 ε 1 ε
U (f, P ) = 1 − −0 +1 + − − +0 1− + = +
2 4 2 4 2 4 2 4 2 4
[( ) ] [( ) ( )] [ ( )]
1 ε 1 ε 1 ε 1 ε 1 ε
L(f, P ) = 1 − −0 +0 + − − +0 1− + = −
2 4 2 4 2 4 2 4 2 4
ε
U (f, P ) − L(f, P ) = < ε.
2
{ }
1
Case ε ≥ 1. Choose P = 0, , 1 . Then
2
( ) ( )
1 1 1
U (f, P ) = 1 −0 +0 1− =
2 2 2
( ) ( )
1 1
L(f, P ) = 0 −0 +0 1− =0
2 2
1
U (f, P ) − L(f, P ) = < 1 ≤ ε.
2
3. If the upper and lower integrals of f on [a, b] are equal, we define the integral of f on [a, b]
to be the common value
∫ b ∫ b ∫ b
f (x) dx := (U ) f (x) dx = (L) f (x) dx.
a a a
160 CHAPTER 7. INTEGRABILITY ON R
Solution. By Theorem 7.1.10, for any partition of [a, b], we have U (f, P ) = L(f, P ) = α(b − a).
It follows that
∫ b
(U ) f (x) dx = inf U (f, P ) = α(b − a),
a P
∫ b
(L) f (x) dx = sup L(f, P ) = α(b − a).
a P
Solution. By Example 7.1.16, for any partition of [a, b], we have U (f, P ) = 1 and L(f, P ) = 0. It
follows that
∫ b
(U ) f (x) dx = inf U (f, P ) = 1,
a P
∫ b
(L) f (x) dx = sup L(f, P ) = 0.
a P
Theorem 7.1.20 If f : [a, b] → R is bounded, then its upper and lower integrals exist and are
finite, and satisfy ∫ ∫
b b
(L) f (x) dx ≤ (U ) f (x) dx.
a a
Theorem 7.1.21 Let a, b ∈ R with a < b, and f : [a, b] → R be bounded. Then f is integrable on
[a, b] if and only if ∫ ∫
b b
(L) f (x) dx = (U ) f (x) dx.
a a
By definition,
∫ b ∫ b
L(f, P ) ≤ (L) f (x) dx and (U ) f (x) dx ≤ U (f, P ).
a a
∑
n
We obtain L(f, P ) = mj (f )∆xj = 0 which is not depend on ε. So,
j=1
∫ 2
(L) f (x) dx = sup L(f, P ) = 0.
0 P
Case 1 ∈
/ P . Then 1 ∈ [xk−1 , xk ] for some k ∈ {1, 2, ..., n}. We have
We obtain
∑
n
ε ε
U (f, P ) − L(f, P ) = U (f, P ) − 0 = Mj (f )∆xj = 3(xk − xk−1 ) < 3 · = < ε.
j=1
6 2
Thus, f is integrable on [0, 2] and
∫ 2 ∫ 2
f (x) dx = (L) f (x) dx = 0.
0 0
7.1. RIEMANN INTEGRAL 163
Solution. Let ε > 0. Let P = {x0 , x1 , ..., xn } be a partition of [0, 1] such that ∥P ∥ < ε.
Y
2
1
X
0 1
We see that
From ∥P ∥ < ε, it follows that |xj − xj−1 | < ε for all j = 1, 2, ..., n. We obtain
Exercises 7.1
∫ 1
1. For each of the following, compute U (f, P ), L(f, P ), and f (x) dx, where
0
{ }
2 1 3
P = 0, , , , 1 .
5 2 5
Find out whether the lower sum or the upper sum is better approximation to the integral.
Graph f and explain why this is so.
3. For each of the following functions, use Pn in 2. to find∫ formulas for the upper and lower
1
sums of f on Pn , and use them to compute the value of f (x) dx.
0
3.1 f (x) = x
1 if 0 ≤ x < 1
2
3.3 f (x) =
0
3.2 f (x) = x 2 if 1
2
≤x≤1
{ }
1
1 if x ∈ E
4. Let E = : n ∈ N . Prove that the function f (x) = is integrable on
n
0 if otherwise
∫ 1
[0, 1]. What is the value of f (x) dx ?
0
∫ c
5. Suppose that f is continuous on an interval [a, b]. Show that f (x) dx = 0 for all c ∈ [a, b]
a
if and only if f (x) = 0 for all x ∈ [a, b].
6. Let f be bounded on a nondegenerate interval [a, b]. Prove that f is integrable on [a, b] if
and only if given ε > 0 there is a partition Pε of [a, b] such that
2. The Riemann sums of f are converge to I(f ) as ∥P ∥ → 0 if and only if given ε > 0 there
is a partition Pε of [a, b] such that
∑
n
P = {x0 , x1 , ..., xn } ⊇ Pε implies f (tj )∆xj − I(f ) < ε
j=1
for all choice of tj ∈ [xj−1 , xj ], j = 1, 2, ..., n. In this case we shall use the notation
∑
n
I(f ) = lim f (tj )∆xj .
∥P ∥→0
j=1
{ }
j
Example 7.2.2 Let f (x) = x2 where x ∈ [0, 1] and P = : j = 0, 1, ..., n be a partition of
n
[0, 1]. Show that if f (ti ) is choosen by the right end point and left end point in each subinterval,
then two I(f ), depend on two methods, are NOT different.
Solution. The Right End Point : Choose f (tj ) = f ( nj ) on the subinterval [xj−1 , xj ]
j j−1 1
and have ∆xj = − = for all j = 1, 2, 3, ..., n. We obtain
n n n
∑ ∑ ( ) n ( )2
1∑ j 1 ∑ 2
n n n
j 1
f (tj )∆xj = f = = 3 j
j=1 j=1
n n n j=1 n n j=1
1 n(n + 1)(2n + 1) (n + 1)(2n + 1)
= 3
· = .
n 6 6n2
Thus,
∑
n
(n + 1)(2n + 1) 1 1
I(f ) = lim f (tj )∆xj = lim 2
= = .
∥P ∥→0
j=1
n→∞ 6n 3 3
166 CHAPTER 7. INTEGRABILITY ON R
∑ ∑ ( ) n ( )2
1 ∑ j−1 1 ∑
n n n
j−1 1
f (tj )∆xj = f = = 3 (j − 1)2
j=1 j=1
n n n j=1
n n j=1
1 [ 2 ]
= 0 + 12 + 22 + · · · (n − 1)2
n3
1 (n − 1)(n)(2(n − 1) + 1) (n − 1)(2n − 1)
= 3 · = .
n 6 6n2
Thus,
∑
n
(n − 1)(2n − 1) 1
I(f ) = lim f (tj )∆xj = lim 2
= .
∥P ∥→0
j=1
n→∞ 6n 3
Theorem 7.2.3 Let a, b ∈ R with a < b, and suppose that f : [a, b] → R is bounded. Then f is
Riemann integrable on [a, b] if and only if
∑
n
I(f ) = lim f (tj )∆xj
∥P ∥→0
j=1
It implies that
∑
n ∫ b
f (tj )∆xj − f (x) dx < ε.
j=1 a
Conversely, assume that the Riemann sums of converge to I(f ). Let ε > 0 and choose a
partition P = {x0 , x1 , ..., xn } of [a, b] such that
∑
n
ε
f (tj )∆xj − I(f ) < (7.2)
j=1
3
for all choices of tj ∈ [xj−1 , xj ]. By the API and APS, choose uj , vj ∈ [xj−1 , xj ] such that
ε ε
Mj (f ) − < f (uj ) and f (vj ) < mj (f ) +
6(b − a) 6(b − a)
It implies that
ε ε ε
f (uj ) − f (vj ) > Mj (f ) − − mj (f ) − = Mj (f ) − mj (f ) − .
6(b − a) 6(b − a) 3(b − a)
So,
ε
Mj (f ) − mj (f ) < f (uj ) − f (vj ) + .
3(b − a)
By (7.2) and telescoping, we have
∑
n
U (f, P ) − L(f, P ) = (Mj (f ) − mj (f ))∆xj
j=1
∑n ∑
n
ε ∑ n
< f (uj )∆xj − f (vj )∆xj + (xj − xj−1 )
j=1 j=1
3(b − a) j=1
∑
n ∑
n
ε
≤ f (uj )∆xj − f (vj )∆xj + (xn − x0 )
j=1 j=1
3(b − a)
∑
n ∑
n
ε
= f (uj )∆xj − I(f ) − f (vj )∆xj + I(f ) + (b − a)
j=1 j=1
3(b − a)
∑
n ∑
n
ε
≤ f (uj )∆xj − I(f ) + f (vj )∆xj − I(f ) +
j=1 j=1
3
ε ε ε
< + + = ε.
3 3 3
Theorem 7.2.4 (Linear Property) If f, g are integrable on [a, b] and α ∈ R, then f + g and αf
are integrable on [a, b]. In fact,
∫ b ∫ b ∫ b
1. (f (x) + g(x)) dx = f (x) dx + g(x) dx
a a a
∫ b ∫ b
2. αf (x) dx = α f (x) dx
a a
∑
n ∫ b ∫ b ∑
n ∑
n ∫ b ∫ b
(f + g)(tj )∆xj − f (x) dx − g(x) dx = f (tj )∆xj + g(tj )∆xj − f (x) dx − f (x) dx
j=1 a a j=1 j=1 a a
∑
n ∫ b ∑
n ∫ b
≤ f (tj )∆xj − f (x) dx + g(tj )∆xj − g(x) dx
j=1 a j=1 a
ε ε
< + = ε.
2 2
∫ b ∫ b ∫ b
We conclude that f + g is integrable on [a, b] and (f (x) + g(x)) dx = f (x) dx + g(x) dx.
a a a
Similarly, if Pε is chosen so that if P = {x0 , x1 , ..., xn } is finer than Pε , then
∑n ∫ b
ε
f (tj )∆xj − f (x) dx < .
j=1 a |α| + 1
∑
n ∫ b ∑
n ∫ b
αf (tj )∆xj − α f (x) dx = |α| f (tj )∆xj − f (x) dx
j=1 a j=1 a
ε
< |α| · < ε.
|α| + 1
∫ b ∫ b
Thus, αf is integrable on [a, b] and αf (x) dx = α f (x) dx.
a a
7.2. RIEMANN SUMS 169
Theorem 7.2.5 If f is integrable on [a, b], then f is integrable on each subinterval [c, d] of [a, b].
Moreover, ∫ ∫ ∫
b c b
f (x) dx = f (x) dx + f (x) dx
a a c
Proof. We may suppose that a < b. Let ε > 0 and choose a partition P of [a, b] such that
Let P0 = P ∪ {c} and P1 = P0 ∩ [a, c]. Since P1 is a partition of [a, c] and P0 is a refinement of P ,
we have
U (f, P1 ) − L(f, P1 ) ≤ U (f, P0 ) − L(f, P0 ) ≤ U (f, P ) − L(f, P ) < ε.
Therefore, f is integrable on [a, c]. A similar argument proves that f is integrable on any subinterval
[c, d] of [a, b].
Let P2 = P0 ∩ [c, d]. Then P0 = P1 ∪ P2 and by definition
Next, we will take infimum of the last inequality over all partitions P of [a, b], we obtain
∫ b ∫ b
f (x) dx = (U ) f (x) dx
a a
∫ c ∫ b
= inf U (f, P ) ≥ f (x) dx + f (x) dx.
P a c
Thus,
∫ a ∫ b ∫ a
f (x) dx = 0 and f (x) dx = − f (x) dx.
a a b
∫ 5
Example 7.2.6 Using the connection between integrals are area, evaluate |x − 2| dx.
0
Y
y = |x − 2|
X
0 1 2 3 4 5
∫ 5 ∫ 5
1 1 13
f (x) dx = ·2·2+ ·3·3= |x − 2| dx =
0 0 2 2 2
∫ 2 √
Example 7.2.7 Using the connection between integrals are area, evaluate 4 − x2 dx.
0
√
Solution. Define f (x) = 4 − x2 where x ∈ [0, 2].
2
√
y= 4 − x2
1
X
0 1 2
∫ 2 ∫ 2 √ 1
f (x) dx = 4 − x2 dx = π(2)2 = π
0 0 4
7.2. RIEMANN SUMS 171
Theorem 7.2.8 (Comparison Theorem) If f, g are integrable on [a, b] and f (x) ≤ g(x) for all
x ∈ [a, b], then ∫ ∫
b b
f (x) dx ≤ g(x) dx.
a a
Proof. Assume that f, g are integrable on [a, b] and f (x) ≤ g(x) for all x ∈ [a, b].
Let P be a partition of [a, b]. By hypothesis, Mj (f ) ≥ Mj (g) whence U (f, P ) ≤ U (g, P ).
It follows that ∫ ∫
b b
f (x) dx = (U ) f (x) dx ≤ U (f, P ) ≤ U (g, P )
a a
for all partition P of [a, b]. Taking the infimum of this inequality over all partition P of [a, b],
we have ∫ ∫ ∫
b b b
f (x) dx ≤ inf U (g, P ) = (U ) g(x) dx = g(x) dx.
a P a a
Theorem 7.2.9 If f is Riemann integrable on [a, b], then |f | is integrable on [a, b] and
∫ b ∫ b
f (x) dx ≤ |f (x)| dx.
a a
Proof. Assume that f is Riemann integrable on [a, b]. Let P = {x0 , x1 , ..., xn } be a partition of
[a, b] and let x, y ∈ [xj−1 , xj ] for j = 1, 2, ..., n. If f (x), f (y) have the same sign, say both are
positive, then
|f (x)| − |f (y)| = f (x) − f (y) ≤ Mj (f ) − mj (f ).
It implies that
Since (7.3) implies that U (|f |, P ) − L(|f |, P ) ≤ U (f, P ) − L(f, P ), it follows that
Thus, |f | is Riemann integrable on [a, b]. Since −|f (x)| ≤ f (x) ≤ |f (x)| holds for any x ∈ [a, b],
we conclude by Theorem 7.2.8 that
∫ b ∫ b ∫ b
− |f (x)| dx ≤ f (x) dx ≤ |f (x)| dx.
a a a
∫ b ∫ b
Hence, f (x) dx ≤ |f (x)| dx.
a a
7.2. RIEMANN SUMS 173
Exercises 7.2
1. Using the connection between integrals are area, evaluate each of the following integrals.
∫ 1 ∫ 2
1.1 |x − 0.5| dx 1.3 (|x + 1| + |x|) dx
0 −2
∫ a √ ∫ b
1.2 a2 − x2 dx, a>0 1.4 (3x + 1) dx, a<b
0 a
∫ b
4. Suppose that gn ≥ 0 is a sequence of integrable function that satisfies lim gn (x) dx = 0.
n→∞ a
∫ b
Show that if f : [a, b] → R is integrable on [a, b], then lim f (x)gn (x) dx = 0.
n→∞ a
∫ 1
5. Prove that if f is integrable on [0, 1], then lim xn f (x) dx = 0.
n→∞ 0
7. Let f be continuous on a closed, nondegenerate interval [a, b] and set M = sup |f (x)|.
x∈[a,b]
7.1 Prove that if M > 0 and p > 0, then for every ε > 0 there is a nondegenerate on interval
I ⊂ [a, b] such that
∫ b
(M − ε) |I| ≤ p
|f (x)|p dx ≤ M p (b − a).
a
(∫ b ) p1
7.2 Prove that lim |f (x)|p dx = M.
p→∞ a
174 CHAPTER 7. INTEGRABILITY ON R
df
Define a set C 1 [a, b] = {f : [a, b] → R : f is differentiable and f ′ are continuous } and f ′ (x) = .
dx
∫ x
Proof. Assume that f is continuous on [a, b] and F (x) = f (t) dt where x ∈ [a, b].
a
Let x0 ∈ [a, b). Then f (x) → f (x0 ) as x → x+
0 . Let ε > 0. There is a δ > 0 such that
F (x0 + h) − F (x0 ) 1 1 1
− f (x0 ) = F (x0 + h) − F (x0 ) − f (x0 ) · h
h h h h
∫ x0 +h ∫ x0 ∫
1 1 1 x0 +h
= f (t) dt − f (t) dt − f (x0 ) dt
h a h a h x0
∫ ∫ ∫ ∫
1 x0 1 x0 +h 1 x0 1 x0 +h
= f (t) dt + f (t) dt − f (t) dt − f (x0 ) dt
h a h x0 h a h x0
∫
1 x0 +h
= (f (t) − f (x0 )) dt
h x0
F (x0 + h) − F (x0 )
Thus, F ′ (x0 ) = lim = f (x0 ). The proof of part 1 is complete.
x→x0 h
7.3. FUNDAMENTAL THEOREM OF CALCULUS 175
2. Assume that f is differentiable on [a, b] and f ′ is integrable on [a, b]. Let ε > 0 and choose
a partition P = {x0 , x1 , ..., xn } of [a, b] such that
∑
n ∫ b
′
f (tj )∆xj − f ′ (x) dx < ε
j=1 a
for any choice of points tj ∈ [xj−1 , xj ]. Use the MVT to choose points tj ∈ [xj−1 , xj ] such that
∫ b ∑
n ∫ b
′
f (b) − f (a) − f (t) dt = (f (xj ) − f (xj−1 )) − f ′ (t) dt
a j=1 a
∑
n ∫ b
′
= f (tj )∆xj − f ′ (t) dt < ε.
j=1 a
∫ b
Thus, f ′ (t) dt = f (b) − f (a) for case x = b. It suffices to prove part 2.
a
Example 7.3.2 Assume that f is differentiable on (0, 1) and integrable on [0, 1]. Show that
∫ 1
xf ′ (x) + f (x) dx = f (1).
0
Proof. Let α ̸= −1. The f ′ (x) = xα . By part 2 of the Fundamental Theorem of Calculus, we
obtain this Theorem.
∫ 1
Example 7.3.4 Find integral x2 dx.
0
Thus, F ′ (0) = 1.
INTEGRATION BY PART.
Theorem 7.3.7 (Integration by Part) Suppose that f, g are differentiable on [a, b] with f ′ , g ′
integrable on [a, b], Then
∫ b ∫ b
′
f (x)g(x) dx = f (b)g(b) − f (a)g(a) − f (x)g ′ (x) dx.
a a
Proof. Assume that f, g are differentiable on [a, b] with f ′ , g ′ integrable on [a, b]. By the Product
Rule, (f g)′ (x) = f ′ (x)g(x) + f (x)g ′ (x) for x ∈ [a, b]. It implies that (f g)′ is integrable on [a, b].
Thus, by the part 2 of the Fundamental Theorem of Calculus, we obtain
∫ b ∫ b ∫ b
′ ′
f (x)g(x) dx = (f g) (x) dx − f (x)g ′ (x) dx
a a a
∫ b ∫ b
′
f (x)g(x) dx = f (b)g(b) − f (a)g(a) − f (x)g ′ (x) dx.
a a
∫ π ∫ 2
2
1. x sin x dx 2. ln x dx
0 1
Solution. By the Integration by Part and The Fundamental Theorem of Calculus, we have
∫ π ∫ π ∫ π
2 2
′ π π 2
x sin x dx = x(− cos x) dx = (− cos ) − 0(− cos 0) − (x)′ (− cos x) dx
0 0 2 2 0
∫ π ∫ π
2 2 π
= cos x dx = (sin x)′ dx = sin − sin 0 = 1.
2
∫ 2 ∫0 2 0
∫ 2
ln x dx = (x)′ ln x dx = 2 ln 2 − 1 ln 1 − x(ln x)′ dx
1 1
∫ 2 ∫ 21
1
= 2 ln 2 − x · dx = 2 ln 2 − 1 dx
1 x 1
∫ 2
= 2 ln 2 − (x)′ dx = 2 ln 2 − (2 − 1) = 2 ln 2 − 1.
1
∫ x3
2
Example 7.3.9 Let f (x) = et dt. Use integration by part to show that
0
∫ 1 ∫ 1
2
6 x f (x)dx − 2
2
ex dx = 1 − e.
0 0
∫ 1 ∫ 1
2
6 x f (x)dx = 2 (3x2 )f (x)dx
0
∫0 1
=2 (x3 )′ f (x)dx
(0 ∫ 1 )
3 ′
= 2 1f (1) − 0f (0) − x f (x)dx
0
( ∫ 1 )
2 x6
= 2 f (1) − 3
x (3x e )dx
0
∫ 1
6
= 2f (1) − 6x5 ex dx
0
∫ 1 ∫ 1
x2
(ex )′ dx
6
=2 e dx −
∫0 1 0
2
=2 ex dx − [e − 1]
0
∫ 1 ∫ 1
2
We conclude that 6 x2 f (x)dx − 2 ex dx = 1 − e.
0 0
178 CHAPTER 7. INTEGRABILITY ON R
CHANGE OF VARIABLES.
Proof. Exercise.
∫ 3
√
e x+1
Example 7.3.11 Find √ dx
0 x+1
√ 1
Solution. Let f (x) = ex and ϕ(x) = x + 1 where x ∈ [0, 3]. Then ϕ′ (x) = √ such that
2 x+1
ϕ(0) = 1 and ϕ(3) = 2. It follows that
√
e x+1
f (ϕ(x)) · ϕ′ (x) = √ .
2 x+1
Solution. Let ϕ(x) = x2 where x ∈ [−1, 1]. Then ϕ′ (x) = 2x such that ϕ(−1) = 1 and ϕ(1) = 1.
It follows that
f (ϕ(x)) · ϕ′ (x) = f (x2 ) · 2x.
Example 7.3.13 Let f : [−a, a] → R where a > 0. Suppose f (−x) = −f (x) for all x ∈ [−a, a].
Show that ∫ a
f (x) dx = 0.
−a
Solution. Let ϕ(x) = −x where x ∈ [−a, a]. Then ϕ′ (x) = −1 such that ϕ(−a) = a and
ϕ(a) = −a. It follows by the Change of Variables that
∫ a ∫ a
f (x) dx = −f (x) · (−1) dx
−a −a
∫ a
= f (−x) · ϕ′ (x) dx
∫−aa
= f (ϕ(x)) · ϕ′ (x) dx
−a
∫ ϕ(a)
= f (t) dt
ϕ(−a)
∫ −a
= f (t) dt
a
∫ a
=− f (t) dt.
−a
∫ a ∫ a
Then, 2 f (x) dx = 0. We conclude that f (x) dx = 0.
−a −a
180 CHAPTER 7. INTEGRABILITY ON R
Exercises 7.3
1. Compute each of the following integrals.
∫ 3
∫ e
1.1 |x + x − 2| dx
2 1.4 x ln x dx
−3 1
∫ 4 √ ∫ π
x−1 2
1.2 √ dx 1.5 ex sin x dx
1 x 0
∫ ∫ √
1 1
4x2 − 4x + 1
1.3 (3x + 1)99 dx 1.6 dx
0 0 x2 − x + 3
2. Use First Mean Value Theorem for Integrals to prove the followingversion of the Mean Value
Theorem for Derivatives. If f ∈ C 1 [a, b], then there is an x0 ∈ [a, b] such that
holds for all c ∈ (a, b), prove that f (x) = 0 for all x ∈ [a, b].
Chapter 8
8.1 Introduction
Let {ak }k∈N be a sequence of numbers. We shall call an expression of the form
∑
∞
ak
k=1
1. The partial sums of S of order n are the numbers defined, for each n ∈ N, by
∑
n
sn := ak .
k=1
2. S is said to converge if and only if its sequence of partial sums {sn } to some s ∈ R as
n → ∞; i.e., for every ε > 0 there is an N ∈ N such that
∑
∞
and call s the sum, or value, of the series ak .
k=1
3. S is said to diverge if and only if its sequence of partial sums {sn } does not converge.
182 CHAPTER 8. INFINITE SERIES OF REAL NUMBERS
∞ [
∑ ]
1 1
Example 8.1.2 Prove that − = 1.
k=1
k k+1
∑
∞
Example 8.1.3 Prove that (−1)k diverges.
k=1
∑
∞
It is easy to see that sn does not converge as n → ∞. Hence, (−1)k diverges.
k=1
1
Theorem 8.1.4 (Harmonic Series) Prove that the sequence k
converges but the series
∑
∞
1
diverges.
k=1
k
1 1 1
≤ ≤ .
k+1 x k
It follows that
∑
n n ∫
∑ k+1 ∫ n+1
1 1 1
sn = ≥ dx = dx = ln(n + 1)
k=1
k k=1 k x 1 x
∑
∞
1
We conclude that sn → ∞ as n → ∞, i.e., diverges.
k=1
k
8.1. INTRODUCTION 183
Theorem 8.1.5 (Divergence Test) Let {ak }k∈N be a sequence of real numbers.
∑
∞
If ak does not converge to zero, then the series ak diverges.
k=1
∑
∞
Proof. Assume that ak converges and equals to s. Then
k=1
∑
n
sn = ak and sn → s as n → ∞.
k=1
Since ak = sk+1 − sk ,
lim ak = lim (sk+1 − sk ) = s − s = 0.
k→∞ k→∞
∑
∞
n
By the Divergence Test, it imlplies that diverges.
k=1
n+1
Thus,
∑
∞
(ak − ak+1 ) = lim (am − an+1 )
n→∞
k=m
= am − lim an+1
n→∞
= am − lim ak .
k→∞
184 CHAPTER 8. INFINITE SERIES OF REAL NUMBERS
∑
∞
1
Example 8.1.8 Evaluate the series .
k=1
(k + 1)(k + 2)
∑
∞
1
Then, sn → ∞ as n → ∞. We conclude that √ √ diverges.
k=1
k+1+ k
∑
∞
Theorem 8.1.10 (Geometric Seires) The series xk converges if and only if |x| < 1, in which
k=1
case
∑
∞
x
xk = .
k=1
1−x
∑
∞
Proof. If |x| ≥ 1, then {x } diverges. By The Divergence Test, it implies that
k
xk diverges.
k=1
Case |x| < 1. Then xk → 0 as k → ∞. Since xk − xk+1 = xk (1 − x), we have
xk xk+1
xk = − .
1−x 1−x
∑
∞ ∑
∞
√
−k
1. 2 2. ( 2 − 1)−k
k=1 k=1
∑
∞
Solution. For 1. We have x = 1
2
such that |x| < 1. It implies that 2−k converges and
k=1
∑
∞ ∞ ( )k
∑ 1
−k 1 2
2 = = = 1.
k=1 k=1
2 1− 1
2
∑∞ ∑∞ ( )k
√ −k 1 1 √
Since ( 2 − 1) = √ and √ = 2 + 1 > 1,
k=1 k=1
2−1 2−1
∑∞
√
we conclude that ( 2 − 1)−k diverges.
k=1
∑
∞ ∑
∞
Theorem 8.1.12 Let {ak } and {bk } be a real sequences. If ak and bk are convergent series,
k=1 k=1
then
∑
∞ ∑
∞ ∑
∞ ∑
∞ ∑
∞
(ak + bk ) = ak + bk and (αak ) = α ak
k=1 k=1 k=1 k=1 k=1
for any α ∈ R.
∑
n ∑
n
Proof. Let sn = ak and tn = bk . Assume that sn → s and tn → t as n → ∞. Then
k=1 k=1
∑
n ∑
n
sn + tn = (ak + bk ) and αsn = αak .
k=1 k=1
∑
∞ ∑
∞
Theorem 8.1.13 If ak converges and bk diverges, then
k=1 k=1
∑
∞
(ak + bk ) diverges.
k=1
Proof. Exercise.
186 CHAPTER 8. INFINITE SERIES OF REAL NUMBERS
∑
∞
1 + 2k+1
Example 8.1.14 Evaluate .
k=1
3k
Solution. Use the Geometric Series and Theorem 8.1.12, it implies that
[( ) ( )k ]
∑∞
1 + 2k+1 ∑∞
1 + 2k+1 ∑∞
1
k
2
k
=2+ k
=2+ +2
k=0
3 k=1
3 k=1
3 3
∑∞ ( )k ∑∞ ( )k
1 2
=2+ +2
k=1
3 k=1
3
1 2
1 13
=2+ 3
+2· 3
=2+ +4= .
1− 1
3
1− 2
3
2 2
∑∞
k
Example 8.1.15 Evaluate k
.
k=1
2
k 1 k−1 2k − k − 1 2k k + 1 k k+1
k
− k = k
= k
= k − k = k−1 − k .
2 2 2 2 2 2 2 2
∑∞ ∑∞ [ ( )]
k 1 k k+1
k
= k
+ k−1
− k
k=1
2 k=1
2 2 2
∑∞ ∑∞ [ ]
1 k k+1
= + − k
k=1
2k k=1 2k−1 2
1
k+1
= 2
+ 1 − lim =1+1−0=2
1− 1
2
k→∞ 2k
Then, the first term is telescoping series and the second term is geometric series. Thus,
[ ( k )k ] ∑ ∞ ( ) ∑ ∞ ( )k
∑∞
1 π 2k π 1 1 1
k 2 1− + = k 2 − (k−1) 2 +
k=1
π π π k=1
π π k=1
π
∑ ∞ ( ) ∑ ∞ ( )k
1 1 1
=− (k−1) 2 − k 2 +
k=1
π π k=1
π
1
1
= −1 + lim + π
1 − π1
2
k→∞ π k
1 2−π
= −1 + 0 + = #
π−1 π−1
∑
∞
1
Example 8.1.17 Evaluate the series .
k=2
k2 −1
Exercises 8.1
1. Show that
∑
∞
xn
xk =
k=n
1−x
2. Prove that each of the following series converges and find its value.
√ √
∑
∞
k+1− k ∑
∞
(−1)k+1 + 4 ∑
∞
2.1 √ 2.3 2.5 2k e−k
k=1
k(k + 1) k=1
5k k=0
∑∞ k+1
(−1) ∑∞
3k ∑∞
2k − 1
2.2 2.4 2.6
k=1
πk k=1
7k−1 k=1
2k
3. Represent each of the following series as a telescopic series and find its value.
∑
∞
1
3.1
k=1
(2k − 1)(2k + 1)
∑∞ ( )
k(k + 2)
3.2 ln
k=1
(k + 1)2
∑∞ √ ( ( π )jk )
π 1
3.3 k
1− , where jk = − for k ∈ N
k=1
4 4 k(k + 1)
∑
∞ ∑∞ ( )k ∑
∞
1 1 k+1
5.1 cos 2 5.2 1− 5.3
k=1
k k k=1
k2
k=1
∑
∞
6. Prove that if ak converges, then its partial sums sn are bounded.
k=1
7.1 Suppose that there is an N ∈ N such that |b − bk | ≤ M for all k ≥ N . Prove that
∑
n ∑
N
nb − bk ≤ |bk − b| + M (n − N )
k=1 k=1
b1 + b2 + · · · + bn
→b as n → ∞.
n
n−1 (
∑ )
k
σn := 1− ak
k=0
n
converges to L as n → ∞.
∑
∞
s1 + s2 + · · · + sn
8.1 Let sn = ak . Prove that σn = for each n ∈ N.
k=0
n
∑
∞
8.2 Prove that if ak ∈ R and ak = L converges, then c is Cesàro summable to L.
k=0
∑
∞
8.3 Prove that (−1)k is Cesàro summable to 12 ; hence the converge of 8.2 is false.
k=0
∑
∞
8.4 TAUBER. Prove that if ak ≥ 0 for k ∈ N and ak is Cesàro summable to L, then
k=0
∑
∞
ak = L.
k=0
∑
∞
9. Suppose that {ak } is a decreasing sequence of real numbers. Prove that if ak converges,
k=1
then kak → 0 as k → ∞.
∑
∞
ak ∑∞
ak
10. Suppose that ak ≥ 0 for k large and converges. Prove that lim = 0.
k=0
k j→∞
k=1
j + k
∑
∞ ∑
∞ ∑
∞
11. If and ak converges and bk diverges, prove that (ak + bk ) diverges.
k=1 k=1 k=1
190 CHAPTER 8. INFINITE SERIES OF REAL NUMBERS
INTEGRAL TEST.
Theorem 8.2.1 (Integral Test) Suppose that f : [1, ∞) → R is positive and decreasing on [1, ∞).
∑
∞
Then f (k) converges if and only if
k=1
∫ n
lim f (x) dx < ∞.
n→∞ 1
∑
n ∫ n
Proof. Let sn = f (k) and tn = f (x) dx for n ∈ N. Since f is positive and decreasing on
k=1 1
[1, ∞), f is locally integrable on [1, ∞). For each k ∈ N, we have
∑
n−1 n−1 ∫
∑ k+1 ∑
n−1
f (k + 1) ≤ f (k + 1) dx ≤ f (k)
k=1 k=1 k k=1
∫ n
sn − f (1) ≤ f (k + 1) dx ≤ sn − f (n)
1
sn − f (1) ≤ tn ≤ sn − f (n)
−f (1) ≤ tn − sn ≤ −f (n)
f (n) ≤ sn − tn ≤ f (1)
Thus, {sn } is bounded if and only if {tn } is. Since f is positir, it implies that both sn and tn are
incresing. It follows that from the Monotone Convergence Theorem that sn converges if and only
if tn converges.
8.2. SERIES WITH NONNEGATIVE TERMS 191
∑
∞
1
Example 8.2.2 Use the Integral Test to prove that diverges.
k=1
k
1
Solution. Let f (x) = . Then f is positive and decreasing on [1, ∞). We obtain
x
∫ n ∫ n
1
lim f (x) dx = lim dx
n→∞ 1 n→∞ 1 x
∫ n
= lim (ln x)′ dx
n→∞ 1
= lim (ln n − ln 1) = ∞.
n→∞
∑
∞
1
By the Integral Test, we conclude that diverges.
k=1
k
∑∞
1
Example 8.2.3 Show that converges.
k=1
k2
1
Solution. Let f (x) = . Then f is positive and decreasing on [1, ∞). We obtain
x2
∫ n ∫ n ∫ n
1
lim f (x) dx = lim dx = lim (−x−1 )′ dx
n→∞ 1 n→∞ 1 x2 n→∞ 1
( )
1
= lim − + 1 = 1 < ∞.
n→∞ n
∑∞
1
By the Integral Test, we conclude that 2
converges.
k=1
k
∑
∞
1
Example 8.2.4 Show that converges.
k=1
k2 + 1
1
Solution. Let f (x) = . Then f is positive and decreasing on [1, ∞). We obtain
x2+1
∫ n ∫ n
1
lim f (x) dx = lim dx
n→∞ 1 n→∞ 1 x2 + 1
∫ n
= lim (arctan x)′ dx
n→∞ 1
π π π
= lim (arctan n − arctan 1) = − = < ∞.
n→∞ 2 4 4
∑
∞
1
By the Integral Test, we conclude that converges.
k=1
k2 +1
192 CHAPTER 8. INFINITE SERIES OF REAL NUMBERS
p-SERIES TEST.
∑∞
1
k=1
kp
Proof. If p < 0 or p = 1, then the series diverges. Case p > 0 and p ̸= 1, set f (x) = x−p and
observe that
has a finite limit if and only if 1 − p < 0. It follows from the Integral Test that p-series converges
if and only if p > 1.
∑
∞
2 −2
Example 8.2.6 Find p ∈ R such that kp converges.
k=1
∑
∞
1
Solution. Rewrite the sum which is a p-series. Then the series converges if and only if
k=1
k 2−p2
2 − p2 > 1. It follows that p − 1 < 0 is equivalent to p ∈ (−1, 1)
2
∑∞ ( )
k + 2k
Example 8.2.7 Determine whether converges or not.
k=1
k2k
Solution. Consider
k + 2k 1 1
k
= k+ .
k2 2 k
∑1
∞ ∑ 1
∞
Since diverges (the p-Series Test, p = 1) and k
converge (the geometric series, x = 12 ),
k=1
k k=1
2
we conclude that
∑∞ ( ) ∑ ∞ ( )
1 1 k + 2k
+ = diverges.
k=1
k 2k k=1
k2k
8.2. SERIES WITH NONNEGATIVE TERMS 193
COMPARISON TEST.
∑
∞
Theorem 8.2.8 Suppose that ak ≥ 0 for k ≥ N . Then ak converges if and only if its sequence
k=1
of partial sums {sn } is bounded, i.e., if and only if there exists a finite number M > 0 such that
∑
n
ak ≤ M for all n ∈ N.
k=1
∑
n ∑
∞
Proof. Let sn = ak for n ∈ N. If ak converges, then sn convergess as n → ∞. Since every
k=1 k=1
convergent sequence is bounded by the BCT, sn is bounded. The proof is complete.
0 ≤ ak ≤ bk for all k ≥ M .
∑
∞ ∑
∞
1. If bk < ∞, then ak < ∞.
k=1 k=1
∑
∞ ∑
∞
2. If ak = ∞, then bk = ∞.
k=1 k=1
∑
n ∑
n
0≤ ak ≤ bk
k=M +1 k=M +1
0 ≤ sn − sM ≤ tn − tM .
Since M is fixed, it follows that sn is bounded when tn is, tn is unbounded when sn is. Apply
Theorem 8.2.8, we obtain this Theorem.
194 CHAPTER 8. INFINITE SERIES OF REAL NUMBERS
∑
∞
1 ∑
∞
1
1. 2.
k=1
k3 + 1 k=1
k 3 + 3k
Solution. Since k 3 + 1 > k 3 > 0 and 3k + k 3 > k 3 > 0 for all k ∈ N, we have
1 1 1 1
0< < 3 and 0< < 3.
k3 +1 k k3 +3 k k
∑∞
1
We see that 3
converges by the p-Series Test (p = 3 > 1). It implies by the Comparison Test
k=1
k
that
∑
∞
1 ∑
∞
1
3
and converge.
k=1
k +1 k=1
k3 + 3k
∑∞
1
Example 8.2.11 Determine whether converges or diverges.
k=2
ln k
Solution. Use the MVT to prove that (see 1.10 of Exercise 6.3)
√
ln x ≤ x for all x > 1.
√
It follows that 0 < ln k ≤ k for all k > 1. Then
1 1
0< √ < .
k ln k
∑∞
1
We see that √ diverges by the p-Series Test (p = 1
2
< 1). It implies by the Comparison Test
k=1
k
that
∑∞
1
diverges.
k=2
ln k
8.2. SERIES WITH NONNEGATIVE TERMS 195
Theorem 8.2.12 (Limit Comparison Test) Suppose that ak and bk are positive for lagre k and
an
L := lim
n→∞ bn
∑
∞ ∑
∞
2. If L = 0 and bk converges, then ak converges.
k=1 k=1
∑
∞ ∑
∞
3. If L = ∞ and bk diverges, then ak diverges.
k=1 k=1
ak
Proof. Assume that ak and bk are positive for lagre k and → L as k → ∞.
bk
L
1. Case 0 < L < ∞. Given ε = . There is an N ∈ N such that
2
ak L
k ≥ N implies −L < .
bk 2
L ak L
For each n ≥ N , we have − < − L < , i.e.,
2 bk 2
L 3L
0 < · bk < ak < · bk .
2 2
Hence, part 1 follows immediately from the Comparison Test and Theorem 8.1.12.
Similar arguments establish part 2 and 3.
∑
∞
1
Example 8.2.13 Use the Limit Comparison Test to prove that converge.
k=1
k2 +1
1 1
Solution. Let ak = and bk = 2 . Then
x2 +1 k
ak k2
lim = lim 2 = 1 < ∞.
k→∞ bk k→∞ k + 1
∑∞
1
We see that converges by the p-Series Test (p = 2 > 1). It implies by the Limit Comparison
k=1
k2
Test that
∑∞
1
2
converges.
k=1
k +1
196 CHAPTER 8. INFINITE SERIES OF REAL NUMBERS
∑
∞
k
Example 8.2.14 Determine whether converges or diverges.
k=1
2k 4 +k+3
k 1
Solution. Let ak = and bk = 3 . Then
2k 4 +k+3 k
ak k4 1
lim = lim 4
= < ∞.
k→∞ bk k→∞ 2k + k + 3 2
∑∞
1
We see that 3
converges by the p-Series Test (p = 3 > 1). It implies by the Limit Comparison
k=1
k
∑∞
k
Test that 4
converges.
k=1
2k + k + 3
∑
∞
1
Example 8.2.15 Determine whether √ converges or diverges.
k=1
k+1
1 1
Solution. Let ak = √ and bk = √ . Then
k+1 k
√
ak k
lim = lim √ = 1 < ∞.
k→∞ bk k→∞ k+1
∑∞
1
We see that √ diverges by the p-Series Test (p = 1
2
< 1). It implies by the Limit Comparison
k=1
k
∑∞
1
Test that √ diverges.
k=1
k+1
∑
∞ ∑
∞
By the Limit comparison Test, it implies that sin |ak | converges if and only if |ak | converges.
k=1 k=1
8.2. SERIES WITH NONNEGATIVE TERMS 197
Exercises 8.2
∑
∞ ∑
∞ ∑∞ ( )
k−3 ln k 1
1.1 3
1.3 , p>1 1.5 10 + k −e
k=1
k +k+1 k=1
kp k=1
k
√
∑∞
k−1 ∑∞
1 ∑∞
3k 2 − k
1.2 1.4 √ 1.6
k=1
k2k k=1
k3k−1 k=1
k4 − k2 + 1
∑
∞
1
4. Find all p ≥ 0 such that the following series converges. p
k=1
k ln (k + 1)
∑
∞
ak
5. If ak ≥ 0 is a bounded sequence, prove that converges for all p > 1.
k=1
(k + 1)p
∑
∞ ∑
∞
|ak |
6. If |ak | converges, prove that converges for all p ≥ 0. What happen if p < 0 ?
k=1 k=1
kp
∑
∞ ∑
∞ ∑
∞
7. Prove that if ak and bk coverge, then ak bk also converges.
k=1 k=1 k=1
∑
∞
1
converges.
k=1
(ak + b)q k
∑
∞ ∑
∞
9. Suppose that ak → 0. Prove that ak converges if and only if the series (a2k + a2k+1 )
k=1 k=1
converges.
198 CHAPTER 8. INFINITE SERIES OF REAL NUMBERS
∑
∞
Theorem 8.3.1 (Cauchy Criterion) Let {ak } be a real sequence. Then the infinite series ak
k=1
converges if and only if for every ε > 0, there is an N ∈ N such that
∑
m
m>n≥N imply ak < ε.
k=n
∑
∞
Proof. Let sn represent the sequence of partial sum of ak and set s0 = 0. By the Cauchy’s
k=1
Theorem (Theorem 2.4.5), sn converges if and only if for every ε > 0, there is an N ∈ N such that
∑
m
ak = |sm − sn−1 | < ε.
k=n
∑
∞
Corollary 8.3.2 Let {ak } be a real sequence. Then the infinite series ak converges if and only
k=1
if for every ε > 0, there is an N ∈ N such that
∑
∞
n≥N implies ak < ε.
k=n
Proof. Exercise.
8.3. ABSOLUTE CONVERGENCE 199
ABSOLUTE CONVERGENCE.
∑
∞
Definition 8.3.3 Let S = ak be an infinite series.
k=1
∑
∞
1. S is said to converge absolutely if and only if |ak | < ∞.
k=1
∑
∞
Theorem 8.3.4 A series ak converges absolutely if and only if for every ε > 0 there is an
k=1
N ∈ N such that
∑
m
m>n≥N implies |ak | < ε.
k=n
∑
∞ ∑
∞
Theorem 8.3.5 If ak converges absolutely, then ak converges.
k=1 k=1
∑
∞ ∑
∞
Proof. Assume that ak converges absolutely. Then |ak | converges.
k=1 k=1
Let ε > 0. By Theorem 8.3.4, there is an N ∈ N such that
∑
m
m>n≥N implies |ak | < ε.
k=n
∑
m ∑
m
ak ≤ |ak | < ε.
k=n k=n
∑
∞
By the Cauchy Criterion, we conclude that ak converges.
k=1
200 CHAPTER 8. INFINITE SERIES OF REAL NUMBERS
∑
∞
(−1)k ∑
∞
(−1)k
Example 8.3.6 Prove that converges absolutely but is not.
k=1
k2 k=1
k
Solution. We consider
∑∞
(−1)k ∑∞
1 ∑∞
(−1)k ∑∞
1
2
= and = .
k=1
k k=1
k2 k=1
k k=1
k
Since the first and second series are a p-series such that p = 2 and p = 1, respectively, we obtain the
∑∞
(−1)k
first series converges but the second series is not. We conclude that 2
converges absolutely
k=1
k
∑ ∞
(−1)k
but is not.
k=1
k
LIMIT SUPREMUM.
Definition 8.3.7 The supremum s of the set of adherent points of a sequence {xk } is called the
limit supremum of {xk }, denoted by s := lim sup xk , i.e.,
k→∞
1 (−1)k 3. zk = 1 + (−1)k
1. xk = 2. yk =
k k
lim sup xk = x.
k→∞
Proof. Assume that xk → x as k → ∞. By the Theorem 2.1.18, any subsequence {xkj } also
converges to x. It implies that lim sup xk = x.
k→∞
{ }
k
Example 8.3.11 Evaluate limit supremum of .
k+1
k
Solution. Since lim = 1, we obtain by Theorem 8.3.10 that
k→∞ k + 1
k k
lim sup = lim = 1.
k→∞ k + 1 k→∞ k + 1
202 CHAPTER 8. INFINITE SERIES OF REAL NUMBERS
ROOT TEST.
1
Theorem 8.3.12 (Root Test) Let ak ∈ R and r := lim sup |ak | k .
k→∞
∑
∞
1. If r < 1, then ak converges absolutely.
k=1
∑
∞
2. If r > 1, then ak diverges.
k=1
Proof. 1. Assume that r < 1. Then there is an x ∈ R such that r < x < 1.
∑
∞
We notice that the geometric series xk converges. By Theorem 8.3.9, we have
k=1
1
|ak | k < x for large k.
∑
∞
It follows that 0 < |ak | < x for large k. By the Comparison Test,
k
|ak | converges.
k=1
2. Assume that r > 1. By Theorem 8.3.9, we have
1
|ak | k > 1 for infinitely many k.
It follows that |ak | > 1 for infinitely many k. Then the limit of ak is not zero.
∑∞
By the Divergence Test, ak diverges.
k=1
∞ (
∑ )k
k
Example 8.3.13 Prove that converges absolutely.
k=1
1 + 2k
∞ (
∑ )k
k
By the Root Test, we conclude that converges absolutely.
k=1
1 + 2k
8.3. ABSOLUTE CONVERGENCE 203
∞ (
∑ )k
3 + (−1)k
Example 8.3.14 Prove that diverges.
k=1
2
Solution. We notice that
( )k 1
k
3 + (−1)k 3 + (−1)k
lim sup = lim sup
k→∞ 2 k→∞ 2
∞ (
∑ )k
3 + (−1)k
By the Root Test, we conclude that diverges.
k=1
2
RATIO TEST.
Theorem 8.3.15 (Ratio Test) Let ak ∈ R with ak ̸= 0 for large k and suppose that
ak+1
r := lim
k→∞ ak
exists as an extended real number.
∑
∞
1. If r < 1, then ak converges absolutely.
k=1
∑
∞
2. If r > 1, then ak diverges.
k=1
Proof. 1. Assume that r < 1. Then there is an x ∈ R such that r < x < 1.
∑
∞
We notice that the geometric series xk converges.
k=1
ak+1 ak+1
By Theorem 8.3.10, we have r = lim = lim sup . By Theorem 8.3.9, we obtain
k→∞ ak k→∞ ak
ak+1
<x for large k.
ak
ak+1 xk+1
It follows that < x = k for large k which is equivalent to
ak x
|ak+1 | |ak |
< for large k.
xk+1 xk
|ak |
Then is decreasing and bounded. So, there is an M > 0 such that |ak | ≤ M xk for all k ∈ N.
xk
∑∞ ∑∞
We see that M xk converges. By the Comparison Test, |ak | converges.
k=1 k=1
204 CHAPTER 8. INFINITE SERIES OF REAL NUMBERS
∑
∞
3k
Example 8.3.16 Prove that converges absolutely.
k=1
k!
3k+1 k! 3
lim · k = lim = 0 < 1.
k→∞ (k + 1)! 3 k→∞ k + 1
∑
∞
3k
By the Ratio Test, we conclude that converges.
k=1
k!
∑∞
k!
Example 8.3.17 Prove that diverges.
k=1
kk
(k + 1)k+1 k! (k + 1)k+1
lim · k = lim
k→∞ (k + 1)! k k→∞ (k + 1)k k
(k + 1)k
= lim
k→∞ kk
( )k
k+1
= lim
k→∞ k
( )k
1
= lim 1 + =e>1
k→∞ k
∑∞
k!
By the Ratio Test, we conclude that k
diverges.
k=1
k
8.3. ABSOLUTE CONVERGENCE 205
Exercises 8.3
∑∞ ∑∞ ∑
∞
∑∞ ( )k2
1 1 2k k
1.1 1.2 1.3 1.4
k=1
k! k=1
kk k=1
k! k+1
k=1
2. Decide, using results convered so far in this chapter, which of the following series converge
and which diverge.
∑∞ ∑∞ ( )k ∞ ( )k 2
k2 k+1 ∑ k!
2.1 2.4 2.7
k=1
πk 2k + 3 (k + 2)!
k=1 k=1
∑
∞
∑∞ ( )k2 ∞ (
∑ )k
k! k 3 + (−1)k
2.2 2.5 2.8
k=1
2k k+1 3
k=1 k=1
∑
∞ ∑∞ ( ) ∑
∞
k! 1 (1 + (−1)k )k
2.3 2.6 π− k −1 2.9
k=1
2 + 3k
k
k=1
k k=1
ek
∑
∞
Prove that ak converges absolutely.
k=1
√ ∑
∞
4. Suppose that ak ≥ 0 and ak → a as k → ∞. Prove that
k ak xk converges absolutely for
k=1
all |x| < 1
a
if a ̸= 0 and for all x ∈ R if a = 0.
5. For eachof the following, find all values of p ∈ R for which the given series converges abso-
lutely.
∑
∞
1 ∑
∞
kp ∑
∞
2kp k!
5.1 5.3 5.5
k=2
k lnp k k=1
pk k=1
kk
∑∞
1 ∑∞
1 ∑∞
√
5.2 5.4 √ 5.6 ( k 2p + 1 − k p )
k=2
lnp k k=2
k(k p − 1) k=1
206 CHAPTER 8. INFINITE SERIES OF REAL NUMBERS
∑
∞
Ak = akj
j=1
∑
∞
for each k ∈ N, and suppose that Ak converges.
k=1
(∞ ) (∞ )
∑
∞ ∑ ∑
∞ ∑
6.1 Prove that akj ≤ akj
j=1 k=1 k=1 j=1
(∞ ) ( )
∑
∞ ∑ ∑
∞ ∑
∞
6.2 Show that akj = akj
j=1 k=1 k=1 j=1
∑
∞ ∑
∞
7. Suppose that ak converges absolutely. Prove that |ak |p converges for all p ≥ 1.
k=1 k=1
∑
∞ ∑
∞
8. Suppose that ak converges conditionally. Prove that k p ak diverges for all p ≥ 1.
k=1 k=1
) (
a2
9. Let an > 0 for n ∈ N. Set b1 = 0, b2 = ln , and
a1
( ) ( )
ak ak−1
bk = ln − ln , k = 3, 4, ...
ak−1 ak−2
an
9.1 Prove that r = lim if exists and is positive, then
n→∞ an−1
∑∞ ( ) ∑∞
1 k−1
lim ln(an ) = limn 1− bk = bk = ln r.
n→∞ n→∞
k=1
n k=1
an+1 1
9.2 Prove that if an ∈ R\{0} and → r as n → ∞, for some r > 0, then |an | n → r as
an
n → ∞.
8.4. ALTERNATING SERIES 207
Theorem 8.4.1 (Abel’s Formula) Let {ak }k∈N and {bk }k∈N be real sequences, and for each pair
of integers n ≥ m ≥ 1 set
∑
n
An,m := ak .
k=m
Then
∑
n ∑
n−1
ak bk = An,m bn − Ak,m (bk+1 − bk )
k=m k=m
∑
n ∑
n
ak bk = am bm + ak bk
k=m k=m+1
∑n
= am bm + (Ak,m − Ak−1,m )bk
k=m+1
∑n ∑
n
= am bm + Ak,m bk − Ak−1,m bk
k=m+1 k=m+1
∑
n ∑
n−1
= am bm + Ak,m bk − Ak,m bk
k=m+1 k=m
∑
n−1 ∑
n−1
= am bm + Ak,m bk + An,m bn − Ak,m bk − Am,m bm+1
k=m+1 k=m+1
∑
n−1
= Am,m bm + An,m bn − Am,m bm+1 − Ak,m (bk+1 − bk )
k=m+1
∑
n−1
= An,m bn − Am,m (bm+1 − bm ) − Ak,m (bk+1 − bk )
k=m+1
∑
n−1
= An,m bn − Ak,m (bk+1 − bk )
k=m
Theorem 8.4.2 (Dirichilet’s Test) Let {ak } and {bk } be sequences in R. If the sequence of
∑
n
partial sums sn = ak is bounded and bk ↓ 0 as k → ∞, then
k=1
∑
n
ak bk converges.
k=1
∑
n
Proof. Let sn = ak be bounded. Assume that bk is decreasing and converges to zero.
k=1
There is an M > 0 such that
∑
n
|sn | = ak ≤ M for all n ∈ N.
k=1
∑
n
|An,m | = ak = |sn − sm−1 | ≤ |sn | + |sm−1 | ≤ M + M = 2M.
k=m
∑
n ∑
n−1
ak bk = An,m bn − Ak,m (bk+1 − bk )
k=m k=m
∑
n−1
≤ |An,m ||bn | + Ak,m (bk+1 − bk )
k=m
∑
n−1
≤ 2M |bn | + |Ak,m ||bk+1 − bk |
k=m
∑
n−1
≤ 2M bn + 2M (bk − bk+1 )
k=m
= 2M bn + 2M (bm − bn )
ε
= 2M bm < 2M · = ε.
2M
∑
n
Thus, ak bk converges.
k=1
8.4. ALTERNATING SERIES 209
∑
∞
Moreover, if ak converges, then
k=1
∑
∞
(−1)k ak converges conditionally.
k=1
∑
∞ ∑
∞
k
Proof. Since the partial sums of (−1) are bounded, (−1)k ak converges by Dirichilet’s Test.
k=1 k=1
∑
∞
(−1)k
Example 8.4.4 Prove that converges conditionally.
k=1
k
1 ∑ ∞
Solution. If ak = , we see that ak is decreasing and converges to 0. By AST, we have (−1)k ak
k k=1
∑∞ ∑∞
1
converges. It is clear that |(−1)k ak | = diverges by p-Series Test (p = 1).
k=1 k=1
k
∑∞
(−1)k
We conclude that converges conditionally.
k=1
k
∑
∞
(−1)k
Example 8.4.5 Prove that converges conditionally.
k=2
ln k
1
Solution. Let ak = . Since k + 1 > k > 0, ln(k + 1) > ln k. It implies that
ln k
1 1
< for all k > 1.
ln(k + 1) ln k
∑
∞
(−1)k
Then ak is decreasing and converges to 0. By AST, we obtain converges.
k=2
ln k
∑∞
1 ∑∞
(−1)k
By Example 8.2.11, . We conclude that converges conditionally.
k=2
ln k k=2
ln k
210 CHAPTER 8. INFINITE SERIES OF REAL NUMBERS
∑
∞
sin(kx)
Example 8.4.6 Prove that S(x) = converges for each x ∈ R.
k=1
k
Exercises 8.4
∑
∞ (π ) ∑
∞
sin(kx)
1.1 (−1) k
− arctan k 1.5 , x ∈ R, p > 0
k=1
2 k=1
kp
∑∞
(−1)k k 2 ∑∞
(−1)k 2 · 4 · · · (2k)
1.2 1.6
k=1
2k k=1
k 2 1 · 3 · · · (2k − 1)
∑
∞
(−1)k ∑
∞
(−1)k
1.3 1.7
k=1
k! k=1
ln(ek + 1)
∑∞
(−1) k ∑∞
arctan k
1.4 , p>0 1.8
k=1
kp k=1
4k 3 − 1
2. For each of the following, find all values x ∈ R for which the given series converges.
∑
∞
xk ∑∞
(x + 2)k
2.1 2.4 √
k=1
k k=1
k k+1
∑
∞
x3k ∑
∞
2k (x + 1)k
2.2 2.5
k=1
2k k=1
k!
∑∞ ∞ (
∑ )k
(−1)k xk k(x + 3)
2.3 √ 2.6
k=1
k2 + 1 k=1
cos k
3. Using any test coveredin this chapter, find out which of the following series converge abso-
lutely, which converge conditionally, and which diverge.
√
∑
∞
(−1)k k 3 ∑∞
(−1)k k + 1
3.1 3.5 √
k=1
(k + 1)! k=1
kk k
∑∞
(−1)(−3) · · · (1 − 2k) ∑
∞
(−1)k sin k
3.2 3.6
k=1
1 · 4 · · · (3k − 2) k=1
k!
∑∞
(k + 1)k ∑∞
(−1)k
3.3 , p>e 3.7 √
k=1
pk k! k=1
k2 + 1
√
∑∞
(−1)k k ∑∞
(−1)k ln(k + 2)
3.4 3.8
k=1
k+1 k=1
k
212 CHAPTER 8. INFINITE SERIES OF REAL NUMBERS
∑
∞
4. ABEL’S TEST. Suppose that ak converges and bk ↓ b as k → ∞. Prove that
k=1
∑
∞
ak bk converges.
k=1
∑
∞
bm ak → 0 as m → ∞.
k=m
8.4. ALTERNATING SERIES 213
Index
Interval, 11 Neighborhood, 72
Closed interval, 11
Nested, 61
Open interval, 11
Nested interval property, 61
Interior point, 77
Nonnegative, 5
Intermedaite value theorem (IVT), 112
Norm, 147
Inverse function, 33
One-to-one, 31
Inverse function theorem, 143
Onto, 31
Isolated point, 77
Open, 69
Irrationals, 2
p-series test, 190
L’Hôspital’s rule, 135
Reference
Gerald B. Folland. (1999). Real Analysis Modern Technique and Their Applications.
John Wiley & Sons, Inc., New York.
Halsey L. Royden and Prtrick M. Fitzpatrick. (2010). Real Analysis (Fourth Edition).
Pearson Education, Inc. New Jersey.
Pual Glendinning. (2012). Maths in minutes. Quercus Editions Ltd, London, England.
Vita
Contact me
Email: [email protected]
Website: www.eledu.ssru.ac.th/thanatyod_ja
Office: 1144